Você está na página 1de 77

2

INDEX
01-07-2019 ........................................................................................................................................................... 3

02-07-2019........................................................................................................................................................... 6

03-07-2019...........................................................................................................................................................8

04-07-2019......................................................................................................................................................... 12

05-07-2019 ......................................................................................................................................................... 15

06-07-2019......................................................................................................................................................... 17

09-07-2019........................................................................................................................................................ 20

10-07-2019 ......................................................................................................................................................... 22

11-07-2019 .......................................................................................................................................................... 25

12-07-2019 .........................................................................................................................................................28

15-07-2019 ......................................................................................................................................................... 31

16-07-2019 ......................................................................................................................................................... 34

17-07-2019 ......................................................................................................................................................... 37

18-07-2019 ........................................................................................................................................................ 40

19-07-2019 .........................................................................................................................................................44

20-07-2019.........................................................................................................................................................46

22-07-2019 .........................................................................................................................................................50

23-07-2019 ......................................................................................................................................................... 52

24-07-2019 ......................................................................................................................................................... 55

25-07-2019 ......................................................................................................................................................... 59

26-07-2019 .........................................................................................................................................................62

27-07-2019 .........................................................................................................................................................64

29-07-2019 ......................................................................................................................................................... 67

30-07-2019......................................................................................................................................................... 70

31-07-2019 ......................................................................................................................................................... 74

www.shankariasacademy.com | www.iasparliament.com
3

01-07-2019 1. It is the world‟s largest tribal festival


celebrated in the southern state of
1) Which one of the following is the most Telangana.
appropriate objective of Register of Indigenous
Inhabitants of Nagaland (RIIN)? 2. It was recently granted the UNESCO‟s
intangible cultural heritage of humanity
a. To provide reservation and socio economic tag.
benefits to the indigenous inhabitants of
Nagaland Which of the statement(s) given above is/are
correct?
b. To prevent issuance of indigenous inhabitant
certificates to ineligible persons a. 1 only
c. To provide Schengen type permit to b. 2 only
indigenous inhabitants of Nagaland to travel to c. Both 1 and 2
any north eastern states
d. Neither 1 nor 2
d. None of the above

5) With respect to Dragonfly sometimes seen


2) Red Dot Campaign sometimes seen in the in the news recently, consider the following
news recently is associated with which of the statements:
following?
1. It is a drone to study the suitability for
a. Fire fighting drills in schools microbial life in Saturn‟s largest moon
b. De-stigmatizing Menstrual talks Titan.
c. Promoting blood donation among youths 2. It was launched recently by the National
Aeronautics and Space Administration
d. To check the irrational use of antibiotics (NASA).
Which of the statement(s) given above is/are
3) Consider the following statements with correct?
respect to Special Rhino Protection Force a. 1 only
(SRPF)
b. 2 only
1. It is basically a tiger protection force
named after the rhino since the threat of c. Both 1 and 2
poaching is more for the one-horned d. Neither 1 nor 2
herbivore.
2. It has been raised at the initiative of
National Tiger Conservation Authority 6) Consider the following statements with
(NTCA). respect to ISALEX 19
Which of the statement(s) given above is/are 1. It is a joint security exercise between
correct? India and El Salvador.
a. 1 only 2. The First edition of this exercise is
recently held in Abu Dhabi.
b. 2 only
Which of the statement(s) given above is/are
c. Both 1 and 2 correct?
d. Neither 1 nor 2 a. 1 only
b. 2 only
4) Consider the following statements with c. Both 1 and 2
respect to Ambubachi Mela
d. Neither 1 nor 2

www.shankariasacademy.com | www.iasparliament.com
4

7) Which of the following countries is/are the c. Both 1 and 2


member(s) of the International Security
d. Neither 1 nor 2
Alliance (ISA)?
1. India
10) Consider the following statements
2. U.A.E
1. They are one of the 16 Naga ethnic
3. France
tribes.
4. Singapore
2. They inhabited the Mon district of
Select the correct answer using the codes given Nagaland.
below:
3. They are known as head hunters of
a. 3 and 4 only North East India.
b. 1, 2 and 3 only Identify the tribal community that matches
correctly with the above description?
c. 2, 3 and 4 only
a. Kuki
d. 1, 2, 3 and 4
b. Mikir
c. Garo
8) How is the Real-Time Gross Settlement
(RTGS) system different from the National d. Konyak
Electronic Funds Transfer system (NEFT)?
1. In NEFT, the transactions received up to
a particular time are processed in Answers
batches, whereas in RTGS, the 1. b
transactions are processed continuously
on a transaction by transaction basis.  The Government of Nagaland has decided to
set up the Register of Indigenous
2. RTGS is used for real time transfers of Inhabitants Certificates of Nagaland
funds up to Rs. 2 lakh, Whereas NEFT is (RIIN), with an objective to prevent
used for fund transfers above Rs. 2 lakh. issuance of fake indigenous inhabitant
Which of the statement(s) given above is/are certificates.
correct?  It is a variant of the National Register of
a. 1 only Citizens (NRC) followed in Assam.
b. 2 only 2. b
c. Both 1 and 2  In its endeavour to make ‗Women Health and
Hygiene empowerment‘ the biggest social
d. Neither 1 nor 2 movement of the 21st Century, Sanfe, a
leading feminine Hygiene brand incubated
by IIT Delhi, has launched Red Dot
9) Consider the following pairs Campaign to normalise period pain
1. Tamizh Yeoman – State Flag of Tamil talk for women.
Nadu
 The campaign intends to generate widespread
2. Jammi Chettu – State Tree of Andhra awareness around the discomfort caused by a
Pradesh period pain in the life of women and how it
restricts them from reaching new heights in
Which of the pair(s) given above is/are various walks of life.
correctly matched?
 Red line campaign is to check the
a. 1 only
irrational use of antibiotics.
b. 2 only
3. c

www.shankariasacademy.com | www.iasparliament.com
5

 Special Rhino Protection Force  The International Security Alliance, launched


(SRPF) is basically a tiger protection in 2017 in Abu Dhabi, is an international
force named after the rhino since the threat of working group to confront organised,
poaching is more for the one-horned transnational and extremist crimes.
herbivore.
 The alliance now comprises nine countries, the
 This force has been raised at the initiative United Arab Emirates, Bahrain, Morocco,
of National Tiger Conservation France, Italy, Spain, Senegal, Singapore, and
Authority for the conservation and the Slovak Republic.
protection of Assam‘s pride, one horned rhinos
from poachers.  India is not a member.
8. a
 Recently, an 82-member SRPF was deployed
in Kaziranga National Park (KNP).  National Electronic Funds Transfer (NEFT) is
4. d an electronic fund transfer system in which the
transactions received up to a particular time
 Assam‘s largest religious congregation, the are processed in batches.
Ambubachi mela at Kamakhya temple atop
Nilachal Hills, was recently inaugurated.  Contrary to this, in Real-Time Gross
Settlement (RTGS), the transactions are
 During the mela, the temple door of the processed continuously on a transaction by
Kamakhya will remain closed. transaction basis throughout the RTGS
business hours.
 Kamakhya temple is one of the 51 Shakti
shrines representing the different body parts of  The RTGS system is primarily meant for large
shiva‘s consort sati. value transactions.
 During the four-day event, it is believed that  The minimum amount to be remitted through
the presiding goddess of the temple, Devi RTGS is Rs. 2,00,000/- with no upper or
kamakhya, goes through her annual cycle of maximum ceiling.
menstruation.
 The NEFT system is used for fund transfers of
5. a up to 2 lakh rupees.
 NASA is sending a drone called Dragonfly to  Fund transfer through RTGS and NEFT
explore Saturn's largest moon, Titan. systems is set to become cheaper after the
Reserve Bank of India decided it will not
 The drone is slated to launch in 2026 and
impose any charges on such transactions.
arrive at Titan in 2034.
9. d
 Using propellers, the drone will fly and land on
several spots on the icy moon to  Tamil Yeoman (Cirro-chroa thais) also
study whether it can support microbial known as Tamizh Maravan, a butterfly
life. species endemic to Western Ghats, has been
declared as the State Butterfly of Tamil
 The nuclear-powered mission is part of NASA's Nadu.
competitive New Frontiers program, which
launched the New Horizons spacecraft that  Jammi Chettu is the State Tree
became the first to visit dwarf planet Pluto. of Telangana.
6. b 10. d
 Representatives of 50 law enforcement  Konyak is one of the 16 Naga ethnic
agencies of the International Security tribes and they are inhabited in Nagaland's
Alliance are taking part in the first joint Mon district.
security exercise ISALEX 19 in Abu
Dhabi beginning recently.  They are known for its fierce headhunting
history.
7. c

www.shankariasacademy.com | www.iasparliament.com
6

 Tattoos were earned for taking an enemy's 2. It was launched by the University
head. Grants Commission (UGC).
Which of the statement(s) given above is/are
correct?
02-07-2019
a. 1 only
1) Which of the following is not one of the five
key interventions under the recently b. 2 only
launched Jal Shakti Abhiyan? c. Both 1 and 2
a. Bore well recharge structures d. Neither 1 nor 2
b. Micro irrigation technologies
c. Water conservation and rainwater harvesting 5) Go Tribal Campaign sometimes seen in the
d. Renovation of traditional and other water news recently aims to?
bodies a. Impart vocational training to tribal youths
b. Promote tribal‘s education in global
2) Consider the following statements with universities
respect to Jal Shakti Abhiyan c. Selecting potential tribal youths for Olympic
1. It aims to reduce water pollution by 20- Games
30% by 2024, taking 2017 as base year. d. Promote tribal arts, crafts in global markets
2. It was launched by the Ministry of Jal
Sakthi.
6) With respect to Korean Demilitarized Zone
Which of the statement(s) given above is/are (DMZ), consider the following statements
correct?
1. It divides the Korean Peninsula into the
a. 1 only Democratic People‟s Republic of Korea
b. 2 only on the north and Republic of Korea on
the south.
c. Both 1 and 2
2. It was created in 1949 after the end of
d. Neither 1 nor 2 the Second World War.
Which of the statement(s) given above is/are
3) Recently, a high-power committee headed by correct?
Maharashtra CM Devendra Fadnavis was a. 1 only
formed to recommend for?
b. 2 only
a. Ethical use of Artificial Intelligence
c. Both 1 and 2
b. Smooth transition to electric vehicles
d. Neither 1 nor 2
c. Tranformation of Indian Agriculture
d. Prevention of Droughts in arid regions
7) Consider the following statements with
respect to Inner Line Permit (ILP)
4) Consider the following statements with 1. It is an official travel document
respect to STRIDE Scheme required by Indian citizens residing
1. This scheme aimed at strengthening outside certain protected states while
research culture and innovation in entering them.
universities and colleges of the country. 2. It is issued by the Government of India
and is obligatory for all those who
reside outside the protected states.

www.shankariasacademy.com | www.iasparliament.com
7

Which of the statement(s) given above is/are 3. Pevek – North Korea


correct?
Which of the pair(s) given above is/are
a. 1 only correctly matched?
b. 2 only  a. 1 only
c. Both 1 and 2  b. 1 and 2 only
d. Neither 1 nor 2  c. 1 and 3 only
 d. 2 and 3 only
8) Consider the following statements with
respect to Akademik Lomonosov
1. It is the world‟s first floating solar
Answers
power station. 1. b
2. It was launched by Russia. There are five key intervention areas under the
movement including
3. Its main aim is to supply power for
research works in Antarctic Regions. 1. Water conservation and rainwater harvesting
Which of the statement(s) given above is/are 2. Renovation of traditional and other water
correct? bodies
a. 2 only 3. Reuse
b. 1 and 2 only 4. Borewell recharge structures
c. 1, 2 and 3 5. Watershed development and intensive
Afforestation
d. None
2. b

9) With respect to Child Labour (Prohibition  Ministry of Jal Shakti has recently
and Regulation) Amendment Act, 2016, launched the Jal Shakti Abhiyan to create
consider the following statements: awareness on the benefits of water
conservation and declining sources of water in
1. It provides for complete prohibition of the country.
employment of children below age 14 in
any occupation without any exceptions.  It has been launched in two phases.

2. The number of violations and  Under the campaign, focus will be on 256
convictions under the Act has seen a water stressed districts and 1,592 water
steady decline since 2014. stressed blocks.

Which of the statement(s) given above is/are  There is no additional funding or specific
correct? targets for the campaign to achieve.
a. 1 only 3. c
b. 2 only  Maharashtra CM Devendra Fadnavis will head
a new high-powered committee to
c. Both 1 and 2 recommend structural reforms in Indian
d. Neither 1 nor 2 agriculture, including changes to the way
farmers market their produce, and norms for
contract farming and essential commodities.
10) Consider the following pairs  It will suggest measures to attract private
1. Kushiro – Japan investment and modernise the agricultural
market system.
2. Khartoum – Syria

www.shankariasacademy.com | www.iasparliament.com
8

 The creation of the panel was originally  Akademik Lomonosov often dubbed as
announced by Prime Minister Narendra Modi the “Floating Chernobyl” is the world‟s
at last month‘s Niti Aayog Governing Council only floating Nuclear Power Station.
meeting.
 Russia is planning to tow this Nuclear power
4. c station to the Arctic port town of Pevek,
Russia.
 University Grants Commission (UGC) has
recently launched the Scheme for Trans-  Upon connection to the grid, it will become the
disciplinary Research for India‘s Developing world's so far only operational floating nuclear
Economy‘ (STRIDE) scheme. power plant and the northernmost nuclear
installation in the world.
 It aimed at strengthening research culture and
innovation in universities and colleges of the 9. b
country.
 The Act stated that, No child shall be
 It will provide support to research projects that employed or permitted to work in any
are socially relevant, locally need-based, occupation or process.
nationally important and globally significant.
 But it exempts the following:
5. d
1. A child helps his family or family enterprise,
 Organised by the Tribal Cooperative Marketing which is other than any hazardous occupations
Development Federation of India (TRIFED), or processes set forth in the Schedule, after his
the 'Go Tribal' campaign will promote and school hours or during vacations.
make available tribal handcrafted textiles,
2. A child works as an artist in an audio-visual
jewellery, and other accessories through global
entertainment industry, including
online market spaces.
advertisement, films, television serials or any
6. a such other entertainment or sports activities
except the circus, subject to such conditions
 The Korean Demilitarized Zone (DMZ) divides and safety measures, as may be prescribed.
the Korean Peninsula into the Democratic
People‘s Republic of Korea on the north and  A recent data from the labour ministry
Republic of Korea on the south. revealed that, the number of violations and
convictions under the Act has seen a steady
 Recently, Donald Trump becomes the first decline since 2014.
serving American President to visit the area.
10. a
 The DMZ was created after the 1953 Korean
War Armistice Agreement, which ended the  Pevek – Russia
Korean War.
 Kushiro - Japan
7. c
 Khartoum – Sudan
 Inner Line Permit (ILP) is an official travel
document required by Indian citizens residing
outside certain ―protected‖ states while 03-07-2019
entering them.
1) Which one of the following is not a criterion
 The ILP is issued by the Government of India for declaring a language as a classical
and is obligatory for all those who reside language?
outside the protected states.
a. High antiquity of its early texts/recorded
 With the ILP, the government aims to regulate history over a period of 1,500-2,000 years
movement to certain areas located near the
international border of India. b. A body of ancient literature/texts which is
considered a valuable heritage by generations
8. a of speakers

www.shankariasacademy.com | www.iasparliament.com
9

c. Should be speak at least by a population not 2. Under the Article, the power to make
less than 2 lakhs and have minimum two changes in the Scheduled Caste list lies
generational speakers only with the Parliament.
d. A literary tradition that is original and not Which of the statement(s) given above is/are
borrowed from another speech community correct?
a. 1 only
2) Consider the following statements with b. 2 only
respect to E-Foreigners Tribunals (e-FTS)
c. Both 1 and 2
1. It is a platform which digitises records
d. Neither 1 nor 2
of all native people living in the state of
Assam.
2. The platform will store biometrics such 5) With respect to RBI Board, consider the
as fingerprints and iris scans. following statements:
Which of the statement(s) given above is/are 1. It consists of both official and non-
correct? official directors.
a. 1 only 2. Official directors include the Governor
and not more than four Deputy
b. 2 only
Governors.
c. Both 1 and 2
3. The RBI Governor has to call a Board
d. Neither 1 nor 2 meeting at least six times in a year, and
at least once each quarter.
Which of the statement(s) given above is/are
3) Consider the following statements with
correct?
respect to Cooperative Sector Exports
Promotion Forum (CSEPF) a. 3 only
1. It aims to achieve the target of doubling b. 1 and 2 only
farm exports to USD 60 billion by 2022.
c. 2 and 3 only
2. It will be set up under the National
d. 1, 2 and 3
Cooperative Development Corporation
(NCDC).
Which of the statement(s) given above is/are 6) Consider the following statements with
correct? respect to the territory of Hong Kong
a. 1 only 1. It is a former British colony located in
the Pacific Ocean.
b. 2 only
2. It is being governed by China under the
c. Both 1 and 2
principle of Two State Solution.
d. Neither 1 nor 2
Which of the statement(s) given above is/are
correct?
4) Consider the following statements with a. 1 only
respect to Article 341 of the Indian
b. 2 only
Constitution
c. Both 1 and 2
1. President, in consultation with the
governor of a state concerned may d. Neither 1 nor 2
notify a caste as a Scheduled Caste.

www.shankariasacademy.com | www.iasparliament.com
10

7) Consider the following statements with Which of the statement(s) given above is/are
respect to National Centre for Polar and Ocean correct?
Research (NCPOR)
a. 1 only
1. It is an autonomous research
b. 2 only
organisation works under the Ministry
of Science and Technology. c. Both 1 and 2
2. It is the nodal agency for the d. Neither 1 nor 2
implementation of the Indian Antarctic
Programme including the maintenance
of India‟s permanent station in Answers
Antarctica.
1. c
Which of the statement(s) given above is/are
correct? The criteria for declaring a language as classical
mandates:
a. 1 only
1. High antiquity of its early texts/recorded
b. 2 only history over a period of 1,500-2,000 years.
c. Both 1 and 2 2. A body of ancient literature/texts which is
d. Neither 1 nor 2 considered a valuable heritage by generations
of speakers.
3. A literary tradition that is original and not
8) ―Himansh‖ sometimes seen in the news borrowed from another speech community.
recently is a/an?
4. Since the classical language and literature is
a. Nuclear powered submarine distinct from the modern, there can also be a
b. High altitude research station discontinuity between the classical language
and its later forms or its offshoots.
c. Wild life conservation centre
 A proposal for granting Marathi the status of
d. None of the above a classical language was under “active
consideration” and the proposal is placed
before a committee of linguistic experts for
9) Working on a Warmer Planet - The Impact consideration.
of Heat Stress on Labour Productivity and
Decent Work is a report published recently by? 2. b

a. International Labour Organisation  The government is enabling a platform to


digitise records of all persons who will be
b. World Economic Forum declared foreigners by the Foreigners
c. UN Framework Convention on Climate Change Tribunals (FTs) in Assam after the final
National Register of Citizens (NRC) is
d. None of the above published.
 The E-Foreigners Tribunals (e-FTs) will
10) With respect to Kisan Credit Card store biometrics such as fingerprints and
(KCC), consider the following statements: iris scans of such persons, and help the
External Affairs Ministry expedite deportation
1. It is available only for agricultural in appropriate cases by reducing paperwork.
farmers and not available for fisheries
and animal husbandry farmers. 3. c
2. The farmers eligible under the KCC  Union Commerce Minister has recently
scheme include small farmers, marginal announced formation of a Cooperative
farmers, share croppers, oral lessee and Sector Exports Promotion Forum
tenant farmers. (CSEPF) to achieve the target of

www.shankariasacademy.com | www.iasparliament.com
11

doubling farm exports to USD 60  Recently, protesters in Hong Kong have


billion by 2022. broken into the LegCo (Legislative Council),
the Parliament building of Hong Kong and
 It will be set up under the National
vandalised it.
Cooperative Development Corporation
(NCDC) which will work with 20 states and  This was in protest against the now-
union territories in the area of export. suspended Extradition bill that would allow
people to be sent to mainland China to face
4. c
trial in courts controlled by the Communist
Under Article 341: Party.
 The President may with respect to any State or 7. b
Union territory, and where it is a State
 The National Centre for Polar and
after consultation with the
Ocean Research (NCPOR) was established
Governorthereof, by public notification,
as an autonomous Research and Development
specify a caste as a Scheduled Caste.
Institution under the Ministry of Earth
 Parliament may by law include in or Sciences.
exclude from the list of Scheduled Castes
 With a mission mandate that is quite
specified in a notification issued under 341 (1)
challenging, the Centre is designated as the
5. d nodal organization for the co-ordination
and implementation of the Indian
 According to the RBI, the ―general Antarctic Programme, including
superintendence and direction of the affairs the maintenance of India‘s permanent
and business of the RBI is entrusted to the station in Antarctica.
Central Board‖ and the Board exercises all
powers and does all acts and things that are  The NCPOR has also been doing research in
exercised by the RBI. the Arctic region and the Antarctica continent.
 The Board consists of official directors, who 8. b
include the Governor and up to four Deputy
 Six Himalayan glaciers monitored by
Governors, non-official directors, who
the National Centre for Polar and Ocean
include up to ten directors from various fields
Research (NCPOR) have been retreating in
and two government officials, and one director
the range of 13 to 33 millimetres per year.
from each of four local boards of the RBI.
 The NCPOR has established a high altitude
 The Governor has to call a Board meeting at
research station named 'Himansh' in
least six times in a year, and at least once
Himalaya at 4,000 metres altitude at Sutri
each quarter.
Dhaka, a remote location in Lahaul-Spiti
6. d district of Himachal Pradesh.
 Hong Kong is a former British colony became a 9. a
special administrative region of China in
 The International Labour Organization (ILO)
1997.
has released a report named 'Working on a
 It is governed under the principle of "One Warmer Planet - The Impact of Heat
Country, Two Systems", under which Stress on Labour Productivity and
China has agreed to give the region a high Decent Work'.
degree of autonomy and to preserve its
 According to the report, India is projected to
economic and social systems for 50 years from
lose 5.8% of working hours in 2030 due to
the date of the handover.
global warming, particularly impacting
 Hong Kong is located in China's southern coast agriculture and construction sectors.
and is surrounded by the South China
 It stated that working hours will be lost
Sea on all sides except the north.
because it is too hot to work or because
workers work at a slower pace.

www.shankariasacademy.com | www.iasparliament.com
12

10. b 1. Constitutional Amendment is required


to change the name of a State.
 Government has extended the facility of Kisan
Credit Card (KCC) to fisheries and animal 2. State‟s approval is not mandatory to
husbandry farmers to help them meet their change the name of a State.
working capital needs.
Which of the statement(s) given above is/are
 The KCC facility will help fisheries and animal correct?
husbandry farmers to meet their short term a. 1 only
credit requirements of rearing of animals,
poultry birds and other aquatic organisms and b. 2 only
capture of fish. c. Both 1 and 2
d. Neither 1 nor 2
04-07-2019
1) Consider the following statements with 4) Consider the following statements with
respect to Gangetic river dolphins respect to International Atomic Energy
1. They were found in India, Bangladesh Agency (IAEA)
and Nepal. 1. It was one among the specialized
2. It is blind and finds its way and prey in agencies of the United Nations (UN).
the river waters through echoes. 2. It was established in response to India‟s
3. It has been listed as endangered under first nuclear operation held in 1974 code
the IUCN Red List. named as “Smiling Budha”.

Which of the statements given above are 3. India joined IAEA in 1993 after the
correct? Second Pokhran Test which was held in
1992.
a. 1 and 2 only
Which of the statement(s) given above is/are
b. 2 and 3 only correct?
c. 1 and 3 only a. 1 only
d. 1, 2 and 3 b. 1 and 3 only
c. 2 and 3 only
2) Consider the following statements with d. None
respect to Adjournment Motion
1. It only terminates a sitting of the House
and not a session of the House. 5) State of the Education Report for India:
Children with Disabilities 2019 was published
2. Rajya Sabha is not permitted to make recently by?
use of this device.
a. UNESCO
Which of the statement(s) given above is/are
correct? b. NITI Aayog

a. 1 only c. Union Ministry of Social Justice and


Empowerment
b. 2 only
d. None of the above
c. Both 1 and 2
d. Neither 1 nor 2
6) Tapan Ray Committee sometimes seen in
the news recently was constituted for which of
3) Consider the following statements the following purposes?

www.shankariasacademy.com | www.iasparliament.com
13

a. To suggest for revamping of PM National c. Caspian Sea


Disaster Relief Fund
d. Mediterranean Sea
b. To study issues in the Prevention of Money
Laundering Act
10) With respect to Prime Minister‘s National
c. To review the framework for Core Investment
Relief Fund (PMNRF), consider the following
Companies
statements:
d. None of the above
1. It has been constituted by an Act of
Parliament.
7) Consider the following statements with 2. It was constituted originally to assist
respect to Mahananda River displaced persons from Pakistan.
1. It is a trans-boundary river that flows 3. It consists entirely of public
through India and Bangladesh. contributions and does not get any
budgetary support.
2. It is a tributary of River Ganga.
Which of the statements given above are
3. Gangetic River Dolphins were found in
correct?
this river.
a. 1 and 2 only
Which of the statement(s) given above is/are
correct? b. 1 and 3 only
a. 1 only c. 2 and 3 only
b. 2 only d. 1, 2 and 3
c. 1, 2 and 3
d. None Answers
1. d
8) Consider the following pairs  Gangetic river dolphins fall under Schedule I
Places in News – Country of the Indian Wild Life (Protection) Act and
have been declared an endangered species
1. Tiware dam – China by the International Union for Conservation of
2. Atacama Desert – Brazil Nature (IUCN).

3. Balochistan – Pakistan  The Gangetic river dolphin is one of the four


freshwater dolphin species in the world.
Which of the pair(s) given above is/are
correctly matched?  The other three are found in the Yangtze river
in China, the Indus river in Pakistan and the
a. 3 only Amazon river in South America.
b. 1 and 3 only  The Gangetic river species — found in India,
c. 2 and 3 only Bangladesh and Nepal — is blind and finds
its way and prey in the river waters through
d. 1, 2 and 3 echoes.
 A population of endangered Gangetic river
9) Lampedusa Island sometimes seen in the dolphins has been found for the first time in
news recently is located in which of the the Mahananda river, a tributary of the
following regions? Ganga, in Bihar‟s Kishanganj district.
a. Black Sea 2. b
b. Sea of Azov  Adjournment Motion is introduced in the
Parliament to draw attention of the

www.shankariasacademy.com | www.iasparliament.com
14

House to a definite matter of urgent 6. c


public importance, and needs the support
of 50 members to be admitted.  The Reserve Bank of India (RBI) has set up a
working group to review the regulatory
 As it interrupts the normal business of the and supervisory framework for core
House, it is regarded as an extraordinary investment companies (CIC).
device.
 The group will be headed by Tapan Ray,
 It involves an element of censure against the non-executive chairman, Central Bank of India
government and hence Rajya Sabha is not and former Secretary in the Ministry of
permitted to make use of this device. Corporate Affairs.
 The discussion on an adjournment motion 7. c
should last for not less than two hours and
 Mahananda River is a trans-boundary river
thirty minutes.
flow through northern India and
 Recently, Defence Minister Rajnath Singh has Bangladesh.
suggested to the Member of Parliaments not
 A population of endangered Gangetic river
to move adjournment motions in the
dolphins has been found for the first time in
House if there is no urgency, because
the Mahananda river, a tributary of the
such practice hamper the productivity of
Ganga, in Bihar‟s Kishanganj district.
Parliament.
8. a
 Adjournment is different from the
Adjournment Motion 1. Tiware dam – Maharashtra, India
3. c 2. Balochistan – Pakistan
 The Constitution authorises the Parliament to  The U.S. had recently designated the Baloch
form new states or alter the areas, boundaries Liberation Army (BLA) as a terror
or names of the existing states without organisation.
their consent.
 Tiware dam located in Ratnagiri in
 The Union Minister of the State for Home Maharashtra has been breached recently.
Affairs has recently informed the Rajya Sabha
9. d
that Constitutional Amendment is
required for changing the name of the  A 31 year old German woman Carola Roackete
state in response to the proposal by the was recently arrested and released later in Italy
Government of West Bengal to change its for rescuing nearly 40 migrants and docking
name to Bangla. her vessel in the Lampedusa Island located
4. d in the Mediterranean Sea against the
Italian police in defiance of Italy‘s anti-migrant
 The International Atomic Energy Agency interior minister.
(IAEA) was created in 1957 in response to the
10. c
deep fears and expectations generated by the
discoveries and diverse uses of nuclear  In pursuance of an appeal by the then Prime
technology. Minister, Pt. Jawaharlal Nehru in January,
1948, the Prime Minister‘s National
 It was established as an autonomous
Relief Fund (PMNRF) was established with
organization which is not under direct
public contributions to assist displaced
control of the UN, but reports to both the
persons from Pakistan.
UNGA and Security Council.
 The fund consists entirely of public
 It is widely known as the world‘s ―Atoms for
contributions and does not get any
Peace‖ Organization.
budgetary support.
 India joined IAEA in 1957.
 PMNRF has not been constituted by the
5. a Parliament.

www.shankariasacademy.com | www.iasparliament.com
15

 Prime Minister is the Chairman of 4) Consider the following statements with


PMNRF and is assisted by Officers/ Staff on respect to Jagannatha Temple
honorary basis.
1. Located in Odisha, this temple is
dedicated to Lord Jagannath, a form of
Lord Shiva.
05-07-2019
2. A Nila Chakra is mounted on the top of
1) The main objective of MOSAiC the shikara of this temple.
mission sometimes seen in the news recently
is? Which of the statement(s) given above is/are
correct?
a. To study the impact of emissions created by
Internal Combustion Engines a. 1 only

b. To study the possibility of mining on sea at b. 2 only


inner core Region c. Both 1 and 2
c. To study the impact of climate change in the d. Neither 1 nor 2
Antarctic Region
d. None of the above
5) Which of the following is/are located in
Italy?
2) With respect to Project Manav sometimes 1. Etna
seen in the news recently, consider the
following statements: 2. Vesuvius
1. It aims to connect India‟s super 3. Stromboli
computers with the Indian educational Which of the statement(s) given above is/are
Institutions. correct?
2. It was launched by the Ministry of a. 3 only
Electronics and Information
Technology. b. 1 and 2 only
Which of the statement(s) given above is/are c. 2 and 3 only
correct? d. 1, 2 and 3
a. 1 only
b. 2 only 6) With respect to Parliamentary Privileges,
c. Both 1 and 2 consider the following statements:
d. Neither 1 nor 2 1. It does not extend to the President who
is also an integral part of the
Parliament.
3) Gaddhika is an art form performed by who 2. They were exhaustively codified in the
among the following? Indian Constitution under the Article
a. Irula Tribes – Tamil Nadu 105.

b. Koya Tribes – Odisha Which of the statement(s) given above is/are


correct?
c. Adiya Tribes – Kerala
a. 1 only
d. None of the above
b. 2 only
c. Both 1 and 2
d. Neither 1 nor 2

www.shankariasacademy.com | www.iasparliament.com
16

7) Schizophrenia refers to which of the Observatory for the Study of Arctic


following? Climate, hopes to enhance understanding
of Arctic climate change and its global
a. Mental Disorder
implications, and lead to safer maritime and
b. Pancreatic Cancer offshore operations and better informed policy
development.
c. Thrombotic Disorder
2. d
d. None of the above
 A new human atlas initiative has been
launched to create a unified database of
8) Consider the following statements with molecular network of all the tissues in the
respect to Gibraltar human body and to derive a holistic picture of
working of the human body.
1. It is a Spanish territory located at the
southern tip of the Iberian Peninsula.  The project named Manav has been launched
by the Department of Biotechnology and
2. The strait of Gibraltar connects the
Persistent Systems, a biotechnology company.
Atlantic Ocean and Mediterranean Sea.
3. c
Which of the statement(s) given above is/are
correct?  The Adiya tribesmen of
a. 1 only Wayanad, Kerala have recently started to
perform ‗Naduzhiyal‘ as part of their
b. 2 only ritualistic art form ‗Nadugaddhika‘ in
villages of the hill district for the prosperity of
c. Both 1 and 2
the public.
d. Neither 1 nor 2
 The tribes believe that ‗gaddhika‘ brings
prosperity to the hamlet and wards off evil hill
9) NAPRT1 Gene sometimes seen in the news district for the prosperity of the public.
recently is associated with which of the  There were two types of the art form — Puja
following? Gaddhika and Naadu Gaddhika.
a. Molecular Scissors 4. b
b. Social bonding gene  Lord Jagannath is a form of Lord
c. Resistant against Fall Army Worm Vishnu.

d. None of the above  The Nila chakra is said to face you from
wherever you view it in Puri.
5. d
10) Kheoni Wildlife Sanctuary is located in
which of the following states?  Recently, Stromboli – a volcano erupts in
Sicily in Italy.
a. Maharashtra
 Italy‘s two other active volcanoes are Etna in
b. Madhya Pradesh Sicily and Vesuvius near Naples.
c. Goa 6. a
d. Chhattisgarh
 Parliament, till now, has not made any special
law to exhaustively codify all the privileges.
Answers  They are based on five sources, namely,
1. d 1. Constitutional provisions,

 The MOSAiC mission, which stands 2. Various laws made by Parliament,


for Multidisciplinary drifting

www.shankariasacademy.com | www.iasparliament.com
17

3. Rules of both the Houses, 06-07-2019


4. Parliamentary conventions, and 1) Consider the following statements with
5. Judicial interpretations. respect to Pradhan Mantri Matsya Sampada
Yojana
7. a
1. The Scheme aims to address critical
 Schizophrenia is a severe mental disorder, gaps in value chain including
characterized by profound disruptions in infrastructure modernization,
thinking, affecting language, perception, and traceability, production, productivity,
the sense of self. post-harvest and quality control.
 It often includes psychotic experiences, such as 2. It is being implemented by the Ministry
hearing voices or delusions. of Fisheries, Animal Husbandry and
Dairying.
 It can impair functioning through the loss of
an acquired capability to earn a livelihood, or Which of the statement(s) given above is/are
the disruption of studies. correct?
8. b a. 1 only
 The British overseas territory of b. 2 only
Gibraltar, a limestone outcrop on c. Both 1 and 2
the southern tip of the Iberian
Peninsula, occupies a commanding position d. Neither 1 nor 2
at the western gateway to the Mediterranean
Sea.
2) Obesogens are chemicals that interact with
 Iran has condemned interception of one of its the genes that makes one fatter. Which of the
oil tankers in Gibraltar where authorities said following are obesogen chemicals?
they suspected it was carrying crude to Syria in
violation of EU sanctions. a. Curcumin
9. d b. Genistein

 A recent study identified a gene c. Capsaicin


called NAPRT1 – that encodes an enzyme d. Zingiberene
involved in vitamin B3 metabolism – is
directly linked to schizophrenia.
 Schizophrenia is a severe mental 3) Consider the following statements with
disorder, characterized by profound respect to Calling Attention motion
disruptions in thinking, affecting language, 1. It is introduced in the Parliament by a
perception, and the sense of self. member to call the attention of a
10. b minister to a matter of urgent public
importance.
 The Kheoni sanctuary, located in the
Kannod tehsil of Dewas district, Madhya 2. It can be introduced only in the Lok
Pradesh was established in 1955. Sabha.

 It is home to mostly cheetal, sambhar, blue 3. Like zero hour, it is also an Indian
bull, four-horned antelope, wild boar, barking innovation.
deer and wolves. Which of the statement(s) given above is/are
correct?
 Recently, five tigers, including an adult male,
two adult females and two cubs have been a. 1 and 2 Only
captured for the first time by night-vision
cameras in Madhya Pradesh‟s Kheoni Wildlife b. 2 and 3 Only
Sanctuary. c. 1 and 3 Only

www.shankariasacademy.com | www.iasparliament.com
18

d. 1, 2 and 3
7) Consider the following statements with
respect to Gandhipedia
4) Consider the following statements with
respect to Dastak Campaign 1. Gandhipedia is being developed in order
to sensitise society, particularly the
1. It was launched by the government of
youth.
Uttar Pradesh.
2. It is being developed under Ministry of
2. It aims to eradicate Acute Encephalitis
Social Justice and Empowerment
Syndrome and Japanese Encephalitis.
Which of the above statements are correct?
3. It is done in association with the help of
World Health Organisation. a. 1 only
Which of the statements given above are b. 2 only
correct?
c. Both 1 and 2
a. 1 and 2 Only
d. Neither 1 nor 2
b. 1 and 3 Only
c. 2 and 3 Only
8) Consider the following taxes
d. 1, 2 and 3
1. Securities Transaction tax
2. Angel tax
5) Consider the following statements with
3. Goods and Service tax
respect to New Space India Limited
4. Capital Gains tax
1. It was established to tap the benefits of
research and development carried out Which of the above comes under direct tax?
by Indian Space Research Organisation
(ISRO). a. 1 and 4 Only

2. It will be established based on the Public b. 1 and 2 Only


Private Partnership (PPP) Model. c. 1, 2 and 4 Only
Which of the statement(s) given above is/are d. All of the above
correct?
a. 1 only
9) Pygmy hog is a critically endangered species
b. 2 only found only in?
c. Both 1 and 2 a. Kaziranga National Park
d. Neither 1 nor 2 b. Manas National park
c. Orang National Park
6) Jal Jeevan Mission recently seen in the news d. Dibru-Saikhowa National Park
aims to?
a. Identifying presence of water and life in Mars
10) The ex-officio members of the National
b. Cleaning river Ganga under Ganga Human Rights Commission are:
Rejuvenation Project
1. National commission for STs
c. Creation of local infrastructure for water
sustainability 2. National commission for Women

d. Establishment of sanitary water treatment 3. Chief Central commissioner for


plants Disabled persons

www.shankariasacademy.com | www.iasparliament.com
19

4. National commission for Minorities  Like the zero hour, it is also an Indian
innovation in the parliamentary procedure
5. National commission for SCs
and has been in existence since 1954.
Select the correct answer using the code given
below:  However, unlike the zero hour, it
is mentioned in the Rules of Procedure.
a. 1, 2, 4 and 5 only
4. a
b. 1, 2, 3 and 5 only
 Uttar Pradesh government has launched a
c. 2, 3, 4 and 5 only massive campaign called ‗Dastak‘ to eradicate
d. 1, 2, 3, 4 and 5 deadly Acute Encephalitis Syndrome (AES)
and Japanese Encephalitis (JE) disease.
 As part of campaign, entire state machinery
Answers with help of UNICEF (United Nations
Children‘s Fund) went door to door in 38
1. c
districts affected by JE and AES.
2. b
 For the first time, the Cultural Department will
 According to a study, genes tend to interact join the Health Department in combating the
with obesogens in the environment to make virus in eastern Uttar Pradesh.
the person fatter.
5. a
 Obesogen are chemicals — pesticides,
 A Public Sector Enterprise viz. New Space
phthalates, medicines — that affect body
India Limited (NSIL) has been incorporated as
functions and makes one fat.
a new commercial arm of Department of Space
 Obesogen include chemicals such as: to tap the benefits of the Research &
Development carried out by ISRO.
1. Monosodium glutamate — popularly called
ajinomoto  The Company will spearhead
commercialization of various space products
2. Clozapine, an anti-psychotic including production of launch vehicles,
drug, Genistein — found in soy products like transfer to technologies and marketing of
milk and baby food, space products.
3. Bisphenol A (BPA): Used in plastic and baby 6. c
bottles and in sealing food cans,
 Jal Jeevan Mission will focus in creation of
4. Arsenic: Chemical present in rocks and soil,
local infrastructure for water sustainability –
percolates into groundwater and accumulates
piped water supply for all homes in rural
in crops,
areas.
5. Nicotine: An addictive chemical found in
tobacco,  The newly constituted Jal Shakti ministry will
helm the program.
6. Dichloro-diphenyltrichloroethane or DDT
popular insecticide.  This mission will focus on integrated demand
and supply-side management of water at the
3. c local level, including the creation of local
infrastructure for source sustainability like
 Calling Attention Motion is introduced in the
rainwater harvesting, groundwater recharge
Parliament by a member to call the attention of
and management of household wastewater for
a minister to a matter of urgent public
reuse in agriculture.
importance, and to seek an authoritative
statement from him on that matter. 7. a
 It can be introduced in any house of the  Gandhipedia is being developed
parliament. by National Council for Science
Museums.

www.shankariasacademy.com | www.iasparliament.com
20

 It aims to sensitise youth and society at large 4) With respect to Foreigners Tribunal,
about positive Gandhian values consider the following statements:
under Ministry of Culture.
1. They are quasi-judicial bodies.
8. c
2. Under the tribunal, the onus of proving
9. b one‟s nationality lies upon such person.
 Only one viable population remains in the Which of the statement(s) given above is/are
Manas Tiger Reserve, but even there, threats correct?
due to livestock grazing, poaching, fire, and a. 1 only
tigers persist.
b. 2 only
10. a
c. Both 1 and 2
d. Neither 1 nor 2
09-07-2019
1) What is the main objective of Utkarsh
2022 sometimes seen in the news recently? 5) Prosecco Hills was recently included in the
UNESCO‟s World Heritage Site. Prosecco Hills
a. To clean the most polluted rivers in the state of are located in?
Uttarakhand by 2022
a. Italy
b. To improve regulation and supervision of the
Reserve Bank of India b. Egypt

c. To promote Uttarkashi as one of the top tourist c. Australia


destination around the world d. Switzerland
d. It is a three year action plan to eradicate Acute
Encephalitis Syndrome in the state of Uttar
Pradesh 6) Consider the following statements with
respect to Indira Gandhi National Centre for
the Arts (IGNCA)
2) The Global Study on Homicide 2019 was 1. It aims to serve as a major resource
published recently by which of the following centre for the arts, especially written,
organisations? oral and visual source materials.
a. UN Office on Drugs and Crime 2. It is an autonomous institution works
b. UN Human Rights Council under the Ministry of Culture.

c. UN Relief and Work Agency Which of the statement(s) given above is/are
correct?
d. Amnesty International
a. 1 only
b. 2 only
3) Gliese 3470 b sometimes seen in the news
recently refers to a/an c. Both 1 and 2

a. Star d. Neither 1 nor 2

b. Comet
c. Black Hole 7) Which one of the following is not the
function of Sangeet Natak Akademi (SNA)?
d. Exo-planet
a. It organizes festivals at all over the country
b. It gives grants-in-aid for research,
documentation, and publishing in the
performing arts

www.shankariasacademy.com | www.iasparliament.com
21

c. It documents and records the performing arts  The Reserve Bank of India (RBI) board has
for its audio-visual archive recently finalised a three- year roadmap to
improve regulation and supervision,
d. None of the above
among other functions of the central
bank (RBI).
8) Consider the following statements with  This medium term strategy —
respect to Ecological Succession named Utkarsh 2022 — is in line with the
1. Primary Succession occurs when an global central banks‘ plan to strengthen the
ecological community first enters into a regulatory and supervisory mechanism.
new form of habitat that it has not been 2. a
present in before.
 The Global Study on Homicide 2019 was
2. Secondary Succession occurs in already published recently by the UN Office on
established habitat. Drugs and Crime (UNODC)
3. In Climax stage, an ecosystem becomes  Asia, which accounts for 60% of the global
balanced and there is a little risk of an population, recorded the lowest rate of
interfering event to influence the homicide in 2017 with only 2.3 killings per
environment. 1,00,000 people while the Americas had the
Which of the statements given above is/are highest homicide rate, according to the UN
correct? report.

a. 3 only 3. d

b. 1 and 2 only  Two NASA telescopes — Hubble and Spitzer,


joined their (metaphorical) telescope hands for
c. 2 and 3 only the first time to seek out chemical 'fingerprints'
d. 1, 2 and 3 of an exoplanet 12.6 times the size of Earth.
 The planet in question, called Gliese 3470 b,
is not in our solar system, which is why it
9) The well of Zamzam sometimes seen in the is called an 'exo-planet'.
news recently is located in?
 Gliese 3470 b is almost a cross between planets
a. Israel Earth and Neptune and has a large, rocky core.
b. Jordan
 Scientists thought this planet would have
c. Saudi Arabia heavier elements like oxygen and carbon, too,
but its atmosphere is dominated with
d. Pacific Ocean hydrogen and helium, much closer in
composition to the Sun.
10) Tadoba-Andhari Tiger Reserve is located in 4. c
which of the following states?
 The Foreigners‘ Tribunals are quasi-judicial
a. Jharkhand bodies meant to ―furnish opinion on the
question as to whether a person is or is not a
b. Chhattisgarh foreigner within the meaning of Foreigners
c. Maharashtra Act, 1946‖.
d. Madhya Pradesh  Section 9 of the Foreigners Act says that
―the onus of proving that such person is not
a foreigner or is not a foreigner of such
Answers particular class or description, as the case may
be, shall, not withstanding anything contained
1. b in the Indian Evidence Act, 1872, lie upon
such person‖.

www.shankariasacademy.com | www.iasparliament.com
22

5. a 2) Plan Bee sometimes seen in the news


recently is a/an
 Italy‘s Prosecco hills northeast of Venice,
which have been cultivated for centuries, were a. Project aimed to safeguard Elephants
recently added to the World Heritage list by
b. Alternative plan to combat Climate Change
the UN cultural organisation (UNESCO).
c. Scheme to promote Apiculture in rural areas
6. c
d. None of the above
7. d
 Sangeet Natak Akademi (SNA) organizes
festivals at all over the country, gives grants- 3) Consider the following statements with
in-aid for research, documentation, and respect to Walled city of Jaipur
publishing in the performing arts; organizes 1. It is an exceptional example of a late
and subsidizes seminars and conferences of medieval trade town in South Asia and
subject specialists; documents and records the defined new concepts for a thriving
performing arts for its audio-visual archive. trade and commercial hub.
8. d 2. The historic walled city of Jaipur in
9. c Gujarat was founded in 1727 AD under
the patronage of Sawai Jai Singh II.
 The well of Zamzam is located in Mecca in
Saudi Arabia and many Haj pilgrims bring 3. It has been recently inscribed as the
the holy water from the well for family and UNESCO‟s World Heritage Site.
friends. Which of the statement(s) given above is/are
10. c correct?
a. 1 only

10-07-2019 b. 1 and 2 only

1) Consider the following statements with c. 1 and 3 only


respect to Start up Village Entrepreneurship d. 1, 2 and 3
Program
1. It aims to support entrepreneurs in
rural areas to set up local enterprises. 4) Helicobacter Pylori is a bacterium resides in
the human stomach. It can cause which of the
2. It is a sub component of Deendayal following in humans?
Antyodaya Yojana – National Rural
Livelihoods Mission (DAY-NRLM). a. Ulcers

3. It was implemented by the Ministry of b. Gastritis


Rural Development. c. Stomach Cancers
Which of the statement(s) given above is/are d. All of the above
correct?
a. 1 only
5) With respect to Payments Council of India,
b. 1 and 2 only consider the following statements:
c. 2 and 3 only 1. It is an umbrella organization
d. 1, 2 and 3 responsible for all retail payments in
India.
2. It is an autonomous organisation works
under the aegis of Reserve Bank of
India.

www.shankariasacademy.com | www.iasparliament.com
23

Which of the statement(s) given above is/are 5. Oil Palm


correct?
Select the correct answer using the codes given
a. 1 only below:
b. 2 only a. 1 and 3 only
c. Both 1 and 2 b. 4 and 5 only
d. Neither 1 nor 2 c. 1, 2, 3 and 5 only
d. 1, 2, 3, 4 and 5
6) With respect to Credit Rating Agencies,
consider the following statements:
9) Which of the following is/are Credit Rating
1. They are statutory bodies established Agencies recognised in India?
under the Credit Rating Agencies
1. Brickwork Ratings
Regulations Act, 1999.
2. India Rating and Research Pvt. Ltd
2. Credit rating agencies in India are
regulated by Reserve Bank of India. 3. Credit Analysis and Research limited
Which of the statement(s) given above is/are 4. Credit Rating Information Services of
correct? India Limited
a. 1 only Select the correct answer using the codes given
below:
b. 2 only
a. 1 and 2 only
c. Both 1 and 2
b. 3 and 4 only
d. Neither 1 nor 2
c. 2, 3 and 4 only
d. 1, 2, 3 and 4
7) Consider the following statements:
1. It is located in the state of Himachal
Pradesh. 10) With reference to the Special officer for
linguistic minorities, consider the following
2. It cuts through the Pir Panjal Range.
statements:
3. It links Manali and Leh by road.
1. The reports of Special officer for
Identify the pass that correctly matches with linguistic minorities are placed before
the above description? the Parliament by the President.
a. Zoji la 2. The procedure for removal of Special
officer for linguistic minorities is
b. Nathu La
provided in the Constitution.
c. Banihal La
Which of the statements given above is/are
d. Rohtang Pass correct?
a. 1 only
8) Which of the following are Plantation crops? b. 2 only
1. Arecanut c. Both 1 and 2
2. Cashew d. Neither 1 nor 2
3. Rubber
4. Cocoa

www.shankariasacademy.com | www.iasparliament.com
24

Answers 2013 catering to the needs of the digital


payment industry.
1. d
 The Council was formed inter-alia for the
 Start up Village Entrepreneurship purposes of representing the various
Program aims to support entrepreneurs regulated non-banking payment
in rural areas to set up local enterprises. industry players, to address and help
 SVEP is a sub component of Deendayal resolve various industry level issues and
Antyodaya Yojana – National Rural barriers which require discussion and action.
Livelihoods Mission (DAY-NRLM) of  PCI works closely with the regulators i.e. RBI,
the Ministry of Rural Development. Finance Ministry, etc.
2. a  National Payments Corporation of
 Plan Bee, an amplifying system imitating the India (NPCI), an umbrella organisation for
buzz of a swarm of honey bees to keep wild operating retail payments and
elephants away from railway settlement systems in India, is
tracks has recently earned the Northeast an initiative of Reserve Bank of India
Frontier Railway (NFR) the best innovation (RBI) and Indian Banks‘ Association
award in Indian Railways for the 2018-19 (IBA) under the provisions of the Payment
fiscal. and Settlement Systems Act, 2007, for
creating a robust Payment & Settlement
3. c Infrastructure in India.
 The Walled City of Jaipur, known for its iconic 6. d
architectural legacy and vibrant culture,
has recently made its entry into the  All the credit rating agencies in India are
UNESCO‘s World Heritage Site list. regulated by Securities and Exchange
Board of India (SEBI) Credit Rating
 The historic walled city of Jaipur Agencies Regulations, 1999 of the Securities
in Rajasthan, was founded in 1727 AD under and Exchange Board of India Act, 1992.
the patronage of Sawai Jai Singh II.
 The government is considering a slew of
 It serves as the capital city of the culturally- reforms to separate their review and rating
rich state of Rajasthan. approval functions to ensure there is no
conflict of interest.
 In town planning, it shows an interchange
of ancient Hindu, Mughal and 7. d
contemporary Western ideas that
resulted in the form of the city.  In the given options, Rohtang pass is the
only pass located in Himachal Pradesh.
4. d
8. d
 The bacterium Helicobacter Pylori resides
in the human stomach can cause ulcers, The major plantation crops include:
gastritis and stomach cancers. 1. Coconut
 A recent study has found that these cancer 2. Arecanut
causing gut bacteria is developing antibiotic
resistance to the three commonly used 3. Oil palm
antibiotics such as Metronidazole, 4. Cashew
Levofloxacin and Clarithromycin.
5. Tea
5. d
6. Coffee
 The Payments Council of India was formed
7. Rubber
under the aegis of Internet and Mobile
Association of India (IAMAI) in the year 8. Cocoa(minor plantation crop)
9. d

www.shankariasacademy.com | www.iasparliament.com
25

 There are a total of six credit agencies in India b. 2 Only


viz,
c. 3 Only
1. ICRA Limited
d. 2 and 3 Only
2. Brickwork Rating
3. India Rating and Research Pvt. Ltd
3) Consider the following statements on
4. Credit Analysis and Research limited (CARE) Central Apprenticeship Council.
5. Credit Rating Information Services of India 1. The Central Apprenticeship Council is a
Limited (CRISIL) statutory body
6. Small and Medium Enterprises Rating Agency 2. The council functions under the
of India (SMREA) Ministry of Labour and Employment
 The government is considering a slew of Which of the above statement(s) is/are
reforms to separate their review and rating correct?
approval functions to ensure there is no a. 1 Only
conflict of interest.
b. 2 Only
10. a
c. Both 1 and 2
 The Constitution does not specify the
qualifications, tenure, salaries and allowances, d. Neither 1 nor 2
service conditions and procedure for removal
of the Special officer for linguistic minorities.
4) Kharchi Puja is a festival observed in the
state of
11-07-2019 a. Odisha
1) ASRAAM which recently appeared in news is b. Tripura
a
c. Nagaland
a. welfare mission for the destitute
d. Kerala
b. short range air to air missile
c. Russian submarine
5) Consider the following statements on
d. None of the above measles.
1. Sri Lanka is the fifth country in WHO‟s
2) Consider the following statements with Southeast Asia region to eliminate
regards to locusts. measles

1. Locusts are related to grasshoppers, 2. India leads the South Asian region in
and the two insects behave in the same eradicating measles
manner although they look differently. Which of the above statement(s) is/are not
2. A swarm of locusts (locusts in large correct?
groups) entering from East Asian a. 1 Only
countries is getting to be a menace in
North-East India. b. 2 Only

3. Locust swarms are harmful to c. Both 1 and 2


environment and human beings. d. Neither 1 nor 2
Which of the statement(s) given above is/are
correct?
a. 1 and 2 only

www.shankariasacademy.com | www.iasparliament.com
26

6) Hayabusa2 spacecraft was launched by


a. Russian Aviation and Space Agency or the 10) The Rohingya, often seen in news, is
Roscosmos State Corporation
a. A tribal community in Cambodia
b. NASA
b. A majorly Buddhist group in Myanmar
c. European Space Agency
c. An Indo-Aryan ethnic group
d. Japan Aerospace Exploration Agency
d. None of the above

7) Which of the following statements is/are


correct? Answers
1. Corporate bonds are debt securities 1. b
issued by both private and public The Indian Air Force (IAF) is set to test-fire ASRAAM
corporations from the Russian Su-30 MKI and the Anglo-French
2. A corporate bond does not involve an Jaguar aircraft by the end of this year (2019).
ownership interest in the company,  ASRAAM is the British Advanced Short
unlike when one purchases a company's Range Air-to-Air Missile (ASRAAM).
equity stock
 It is a Within-Visual-Range (WVR) dominance
a. 1 Only weapon with a range of over 25 km.
b. 2 Only
 It accepts target information via aircraft
c. Both 1 and 2 sensors, such as radar or helmet-mounted
sight, but can also act as an autonomous
d. Neither 1 nor 2 infrared search and track system.
2. c
8) Which of the following statement(s) is/are
 Locusts are a certain species of short-horned
not correct?
grasshoppers.
1. Art 370 allows the Jammu and Kashmir
legislature to define the list of  Locusts are related to grasshoppers, and the
„permanent residents‟ of the state two insects look similar but behave
differently. While lifestyle being much like
2. Article 35A provides temporary, grasshoppers, locusts have another behavioral
transitional and special status to phase called the gregarious phase. When
Jammu and Kashmir environmental conditions produce many green
plants and promote breeding, locusts can
a. 1 Only
congregate into thick, mobile, ravenous
b. 2 Only swarms (dense groups).
c. Neither 1 nor 2  There has been an incursion of desert
locusts in Rajasthan and Gujarat from
d. Both 1 and 2
areas bordering Pakistan, recently. They
emerged from Sudan and Eritrea on Africa's
Red Sea Coast and travelled through Saudi
9) Diamond Harbour is a port located in the
Arabia and Iran to enter Pakistan.
state of -
a. West Bengal  Locust swarms devastate crops and cause
major agricultural damage and associated
b. Maharashtra human misery such as famine and
starvation.
c. Odisha
3. a
d. Tamil Nadu

www.shankariasacademy.com | www.iasparliament.com
27

The 36th Central Apprenticeship Council meet was The other four countries are Bhutan, Maldives,
held recently. DPR Korea and Timor-Leste.
 The Central Apprenticeship Council (CAC) is  India has still a long way to go in
an apex statutory body under the Apprentices eradicating measles. The concerns with
Act, 1961. and mistrust in vaccines is a huge challenge in
this process. The Delhi High Court, in 2018,
 The Ministry of Skill Development & had put on hold a vaccination campaign, citing
Entrepreneurship (MSDE) organizes the lack of parental consent.
meetings of the Central Apprenticeship
Council (CAC).  Measles is a deadly childhood infection
(sometimes in adults) and a serious and highly
 The council aims to meet the aspirations of the contagious disease. It can cause debilitating or
Indian youth who strive to gain on-the-job fatal complications, including encephalitis,
training and secure better opportunities for severe diarrhoea and dehydration, pneumonia,
employment. ear infections and permanent vision loss.
 Minister of State for Skill Development and 6. d
Entrepreneurship is the Vice Chairman of the
council.  Hayabusa2 spacecraft was launched by Japan
Aerospace Exploration Agency, or JAXA.
 Other members of the council include -
 JAXA recently confirmed that Hayabusa2
1. representatives of the Establishment spacecraft successfully landed on a distant
in the PSUs, Private Sector asteroid and collected underground samples
2. representatives from Central (soil sample).
Ministries and State /UT Governments
 Hayabusa2 is the first to successfully collect
3. persons having special knowledge and underground soil samples from an asteroid.
experience on the matters relating to
 The asteroid is named Ryugu, after an
industries, labour
undersea dragon palace in a Japanese folktale.
4. representatives of the All India Council It is about 300 million kilometers (180 million
of Technical Education and UGC miles) from Earth.
4. b  The achievement comes ahead of a similar
Kharchi Puja, a centuries-old Puja, was recently mission planned by the US's NASA team at
inaugurated in Tripura‘s erstwhile capital Puran another asteroid.
Habeli. 7. c
 The seven-day Kharchi Puja is an annual In Budget 2019, the Finance Minister has announced
festival meant to cleanse the sins of mortal fresh measures to boost the development of India‟s
souls. corporate bond market.
 Originally a Hindu tribals' festivity, it is now  Corporate bonds are debt securities issued by
observed by all communities and religions. private and public corporations to raise money
for a variety of purposes.
 The worship starts with the dipping of 14
deities in the Howrah river, followed by the  A buyer buys a corporate bond, and lends
sacrifice of 108 animals in the presence of money to the "issuer" i.e. the company that
hundreds of thousands of devotees, all at issued the bond, and gets periodical returns as
government expense. interest until the principal amount is paid at a
specified maturity date.
5. b
The WHO has recently declared the elimination of  Notably, a corporate bond does not involve an
measles from Sri Lanka. ownership interest in the company. This is
unlike the case when one purchases the
 Sri Lanka is the fifth country in WHO‘s company's equity stock, which involves
Southeast Asia region to eliminate measles. ownership terms.

www.shankariasacademy.com | www.iasparliament.com
28

8. d  Thousands of Rohingya have been fleeing


Myanmar, many of whom are moving to
 Article 35A allows the Jammu and Kashmir
Bangladesh and India, which is leading to a
legislature to define the list of ‗permanent
refugee crisis.
residents‘ of the state. It was added to the
constitution through the Constitution
(Application to Jammu and Kashmir) Order,
1954, a presidential order. 12-07-2019
 Art 370 provides temporary, transitional 1) Consider the following statements.
and special status to Jammu and Kashmir. 1. Aadhaar number can be used as a proof
Recently, the government has informed the of identity of a person, only by
Parliament that no foreign government or authentication.
organisation had any locus standi in repealing Article 2. The UIDAI can decide whether an entity
370. The reply came in response to a question on can use Aadhaar or not.
whether repeal of these articles in any way violate UN
regulations or any international obligation of the 3. The fees and revenue collected by the
country. UIDAI goes to the Consolidated Fund of
India
9. a
Which of the following are true according to
 The Diamond Harbour in West Bengal is the recent changes brought through the
located at the mouth of river Hooghly. Aadhaar and Other Laws (Amendment) Bill,
2019?
 It was recently in news as it recorded
the maximum sea level increase, among a. 1 Only
the major ports in India. Diamond Harbour is
b. 2 Only
followed by Kandla port in Gujarat, Haldia in
West Bengal, and Port Blair on South c. 3 Only
Andaman Island.
d. None of the statements is true
 Chennai and Mumbai recorded a sea level rise
far below the global and the national averages.
2) Which of the following statement(s) is/are
10. c
correct?
Former U.N. Secretary-General Ban Ki-moon recently
1. The Union Cabinet has approved a new
expressed concern over risks of monsoon floods
definition for child pornography in its
threatening the lives of Rohingya refugees in camps in
amendments to the POCSO Act
Bangladesh.
2. Child pornography is already defined in
 Rohingya are an Indo-Aryan ethnic group,
Section 67 of the IT Act and Section 293
largely comprising Muslims, who
of the IPC (on sale, etc., of obscene
predominantly live in the Western
objects to young person)
Myanmar province of Rakhine.
a. 1 Only
 They speak a dialect of Bengali, as opposed to
the commonly spoken Burmese language. b. 2 Only
 Though they have been living in the South East c. Both 1 and 2
Asian country for generations, Myanmar d. Neither 1 nor 2
considers them as persons who migrated to
their land during the Colonial rule. So, it has
not granted Rohingyas full citizenship. 3) The Eradi Tribunal was set up to reassess
Sectarian violence between Rohingyas and availability and sharing of water in connection
Rakhine‘s Buddhist natives began flaring up in with
2012.
a. Mahadayi river dispute

www.shankariasacademy.com | www.iasparliament.com
29

b. Satluj-Yamuna Link canal issue d. Ministry of Environment, Forest and Climate


Change
c. Mullaperiyar dam dispute
d. Zangmu dam issue
8) Consider the following statements on
Worker safety code Bill.
4) „Operation Thirst‟ is an initiative of the
1. Worker safety code Bill subsumes
a. Ministry of Railways existing laws regarding minimum wages
and other workers‟ remuneration
b. Ministry of Environment, Forest and Climate issues.
Change
2. It would apply to all establishments with
c. Ministry of Drinking Water and Sanitation 10 or more workers.
d. Ministry of Rural Development 3. It is the first of the four proposed codes
that aim to merge 44 labour laws.
5) Anti-defection law is dealt with in the Which of the above statement(s) is/are not
constitution in the correct?
a. 7th Schedule a. 1 and 2 Only
b. 8th Schedule b. 1 and 3 Only
c. 9th Schedule c. 2 and 3 Only
d. 10th Schedule d. None of the above

6) Which of the following comes under the 9) Which of the following is correct regarding
applications of facial recognition technology? the Central Pollution Control Board (CPCB)?
1. Payments 1. The CPCB was constituted under the
Water (Prevention and Control of
2. Access and security Pollution) Act.
3. Criminal identification 2. The CPCB was entrusted with the
4. Advertising powers and functions under the Air
(Prevention and Control of Pollution)
5. Healthcare Act.
Select the correct answer using the codes given Which of the statement(s) given above is/are
below: correct?
a. 1, 2, and 3 Only a. 1 Only
b. 2 and 3 Only b. 2 Only
c. 4 and 5 Only c. Both 1 and 2
d. All of the above d. Neither 1 nor 2

7) The International Cooperation Scheme is 10) Consider the following statements on


related to which of the following Ministry? Periodic Labour Force Survey (PLFS).
a. Ministry of External Affairs 1. PLFS measures employment every 3
b. Ministry of Human Resource Development months in urban areas and once a year
in both rural and urban areas.
c. Ministry of Micro, Small and Medium
Enterprises

www.shankariasacademy.com | www.iasparliament.com
30

2. The quarterly survey only captures data  The Union Cabinet has approved a new
classed as current weekly status (CWS), definition for child pornography.
while the annual survey measures both
the usual status and CWS.  In line with the Women and Child
Development Ministry‘s initiative on zero
3. The National Sample Survey Office tolerance to child pornography, the Cabinet
(NSSO) under the Ministry of Statistics has approved a new definition for child
and Programme Implementation pornography.
conducts the survey.
 Under this, any visual depiction of sexually
Which of the above statements are correct? explicit conduct involving a child will be
a. 1 and 2 Only punishable.

b. 2 and 3 Only  These include photographs, video, digital or


computer generated image indistinguishable
c. 1 and 3 Only from an actual child and an image created,
d. All the statements are correct adapted or modified but appear to depict a
child. It will also apply to animated content
depicting children and where adults pretend to
be children.
Answers
3. b
1. b
A recent Supreme Court order asked Punjab, Haryana
The Aadhaar and Other Laws (Amendment) Bill, 2019
and Centre to sort out SYL (Satluj-Yamuna Link
was recently passed in the Parliament.
canal) issue amicably.
 Under the Bill, an individual may voluntarily
 Apart from Ravi and Beas, Satluj and
use his/her Aadhaar number to establish
Yamuna are two major rivers that flow
his/her identity, by authentication or
through Punjab and Haryana. The dispute
offline verification. [Authentication –
is about water sharing between two states
submitting Aadhaar number and biometric or
depending on agriculture, with Punjab saying
demographic information to the Central
it has no water to spare.
Identities Data Repository. Offline verification
- other modes specified by the Unique  The Eradi Tribunal headed by Supreme Court
Identification Authority of India (UIDAI)] Judge V Balakrishna Eradi was set up to
reassess availability and sharing of water in
 The existing Act allowed State or a corporate
SYL. The tribunal gave its verdict in 1987.
entity under any law to use Aadhaar. Under
However, the dispute is still unresolved.
the Amendment Bill, the UIDAI can make a
decision on entities using AADHAAR.  Mahadayi river dispute - Karnataka and
Goa
 The Amendment Bill provides for a
dedicated Unique Identification  Mullaperiyar dam dispute - Kerala and
Authority of India Fund, which will Tamil Nadu
receive all fees and revenue collected by the
UIDAI. Earlier, it goes to the Consolidated  Zangmu dam issue - India-China; a dam
Fund of India. built by China on the Brahmaputra river near
Zangmu in Tibet
2. a
4. a
The Union Cabinet has approved amendments in
POCSO Act to make punishment more stringent for  ‗Operation Thirst‘ is an all India drive initiated
sexual crimes against children. by the Ministry of Railways, recently.

 Neither Section 67 of the IT Act nor  The objective is to curb the menace of
Section 293 of the Indian Penal Code unauthorized PDW (packaged drinking water)
defines child pornography. in Railway premises.
5. d

www.shankariasacademy.com | www.iasparliament.com
31

Anti-defection Law appeared in news recently in 8. b


connection with political developments in Karnataka,
The Union Cabinet recently approved the Code on
Goa and some other states.
Wages Bill and the Worker safety code Bill.
 The anti-defection law is contained in the 10th
 The Worker safety code Bill seeks
Schedule of the Constitution.
to merge 13 labour laws into one code
 It was passed through the Constitution on occupational safety, health and
52nd Amendment Act, 1985 and came into working conditions.
effect the same year.
 It is the Code on Wages Bill that subsumes
6. d existing laws regarding minimum wages and
other workers‘ remuneration issues as part of
The National Crime Records Bureau (NCRB) recently
the efforts to improve ease of doing business.
released a Request for Proposal for an Automated
Facial Recognition System (AFRS) to be used by police  Worker safety code Bill would apply to all
officers establishments with 10 or more workers.
 Facial recognition is the process of identifying  Worker safety code Bill is the second of
or verifying the identity of a person using their four proposed codes that aim to merge 44
face. It captures, analyzes and compares labour laws. The first is the Code on Wages
patterns based on the person's facial Bill.
details. The artificial intelligence technology
used for pattern-finding and matching, in this 9. c
regard, is called ―neural networks‖. The CPCB recently pulled up 52 companies for not
specifying a timeline or a plan to collect the plastic
 The AFRS, being implemented by the National
waste that results from their business activities.
Crime Records Bureau (NCRB), is a
component of Crime and Criminal Tracking  Both statements are right.
Network and Systems (CCTNS).
 The Central Pollution Control Board (CPCB) is
 All of the areas given above could be benefited a statutory organisation constituted in 1974
with facial recognition technology. under the Water (Prevention and Control
of Pollution) Act, 1974.
 In advertisement field, the FaceTech helps
companies recognise certain demographics  Further, CPCB was entrusted with the powers
(age and gender), and facilitates marketers and and functions under the Air (Prevention
advertisers better targeting. In healthcare, it and Control of Pollution) Act, 1981
helps medical professionals identify illnesses
by looking at a patient‘s features and better 10. d
streamlining of appointment process. PLFS is a recent initiative aimed at generating
7. c estimates of various labour force indicators. All the
given statements are correct.
The Ministry of Micro, Small and Medium
Enterprises (MSME) is implementing
the International Cooperation (IC) Scheme. 15-07-2019
 The objective is to enhance the competency of 1) Consider the following statements with
MSMEs, capturing new markets for their respect to Vikramshila Gangetic Dolphin
products, exploring new technologies for Sanctuary (VGDS)
improving manufacturing capacity, etc.
1. It is the only dolphin sanctuary in India.
 Under it, financial assistance is provided on
reimbursement basis to the eligible 2. It is located in Bihar on the Ganga
State/Central Government Organisations and River.
Registered Industry Associations, societies 3. National Water Way – 1 passes through
/trusts associated with promotion and this sanctuary.
development of MSME sector.

www.shankariasacademy.com | www.iasparliament.com
32

Which of the statement(s) given above is/are d. Neither 1 nor 2


correct?
a. 1 only
5) Orchids of India: A pictorial guide was
b. 1 and 2 only recently published by which of the following?
c. 2 and 3 only a. Ministry of Earth Sciences
d. 1, 2 and 3 b. Ministry of Science and Technology
c. Ministry of Chemicals and Fertilizers
2) Consider the following statements with d. Ministry of Environment, Forest and Climate
respect to Investigation Tracking System for Change
Sexual Offences (ITSSO)
1. It is an analytics tool that aims to
6) Match the following:
monitor and track time-bound
investigation. 1. Lithophyte – a. plants growing on
another plant including those growing
2. It is a part of the Crime and Criminal
on rock boulders
Tracking Network and System (CCTNS).
2. Terrestrial – b. plants growing on land
Which of the statement(s) given above is/are
and climbers
correct?
3. Mycoheterotrophic – c. plants which
a. 1 only
derive nutrients from mycorrhizal fungi
b. 2 only that are attached to the roots of a
vascular plant
c. Both 1 and 2
Select the correct answer using the codes given
d. Neither 1 nor 2
below:
a. 1-a; 2-b; 3-c
3) Global Multidimensional Poverty Index
b. 1-b; 2-a; 3-c
(MPI) 2019 Report was published recently by
which of the following? c. 1-c; 2-b; 3-a
a. World Economic Forum d. 1-a; 2-c; 3-b
b. UN Inter Agency Group
c. Oxford Poverty and Human Development 7) Consider the following statements with
Initiative respect to Non Communicable Diseases
d. Both B and C 1. About 61 per cent of deaths in India
were attributed to Non Communicable
Diseases (NCDs), including cancer,
4) Consider the following with respect to Rewa tuberculosis and diabetes.
Power Project
2. Almost 23 per cent are at risk of
1. It is Asia‟s largest solar power project. premature death due to Non
Communicable Diseases.
2. It is located in the state of Maharashtra.
Which of the statement(s) given above is/are
Which of the statement(s) given above is/are correct?
correct?
a. 1 only
a. 1 only
b. 2 only
b. 2 only
c. Both 1 and 2
c. Both 1 and 2

www.shankariasacademy.com | www.iasparliament.com
33

d. Neither 1 nor 2  In February 2019, the Home Ministry


launched an analytics tool — Investigation
Tracking System for Sexual Offences
8) Consider the following statements with (ITSSO) to monitor and track time-
respect to Epiphytes bound investigation.
1. These are plants growing on other  It is part of the Crime and Criminal
plants and derive its nutrition source Tracking Network and System
from the supporting plant. (CCTNS) that connects over 15,000 police
2. The occurrence of epiphytic orchids stations across the country.
decreases with the increase in altitude.  ITSSO analysis has recently found out that
Which of the statement(s) given above is/are nearly half of sex offence cases not being
correct? investigated in time.

a. 1 only 3. c

b. 2 only  The UN Development


Programme (UNDP) and the Oxford
c. Both 1 and 2 Poverty and Human Development
d. Neither 1 nor 2 Initiative (OPHI) has recently released the
global Multidimensional Poverty Index (MPI)
report for 2019.
9) “NotPetya” sometimes seen in the news  It has found that Jharkhand has made the
recently is a fastest improvement among Indian states in
a. Ransomware reducing poverty.

b. Crypto currency 4. a

c. Hydrogen Car  In Madhya Pradesh, continuous rain has


damaged Asia's largest solar Power
d. Super Computer project in Rewa.
 The damage was caused after heavy rain
10) Operation Vijay is associated with which of triggered a mudslide.
the following?  As per the estimates, this project has suffered a
a. Annexation of Goa (1961) loss of more than Rs 20 crores by rain.
b. Annexation of Hyderabad (1948) 5. d
c. Indo-Pakistan war of (1971)  Orchids of India: A Pictorial Guide, a
publication detailing all the species of India
d. Pokhran –I nuclear test (1974) was unveiled recently by the Ministry of
Environment, Forest and Climate
Change.
Answers
6. a
1. d
Orchids can be broadly categorised into three life
 The Vikramshila Gangetic Dolphin Sanctuary forms:
(VGDS), from Sultanganj to Kahalganj on the
Ganga in Bihar is the only dolphin sanctuary in 1. Epiphytic (plants growing on another plants
the country. including those growing on rock boulders and
often termed lithophyte)
 The Sultanganj-Kahalgaon stretch of National
2. Terrestrial (plants growing on land and
Waterway-1 passes through this sanctuary.
climbers)
2. c

www.shankariasacademy.com | www.iasparliament.com
34

3. Mycoheterotrophic (plants which derive 16-07-2019


nutrients from mycorrhizal fungi that are
attached to the roots of a vascular plant) 1) Indrasaurus wangi sometimes seen in the
news recently is a/an
7. b
a. Paravian Dinosaur
 Tuberculosis is a communicable disease.
b. Ancient Lizard species
8. b
c. Dolphin native to River Indus
Orchids can be broadly categorised into three life
forms: d. None of the above

1. Epiphytic (plants growing on another plants


including those growing on rock boulders and 2) Consider the following statements with
often termed lithophyte) respect to Blue Flag Programme
2. Terrestrial (plants growing on land and 1. It is an eco-label awarded to beaches,
climbers) marinas, and sustainable boating
3. Mycoheterotrophic (plants which derive tourism operators.
nutrients from mycorrhizal fungi that are 2. It is jointly operated under the auspices
attached to the roots of a vascular plant) of the International Maritime
 Epiphyte, also called air Organisation and UN-Oceans.
plant or Lithophyte, are any plant that 3. Shivrajpur and Bhogave beaches are the
grows upon another plant or object merely for only beaches in India that have attained
physical support. the Blue Flag.
 Epiphytes have no attachment to the ground or Which of the statement(s) given above is/are
other obvious nutrient source and are not correct?
parasitic on the supporting plants.
a. 1 only
 The epiphytic orchids are abundant up to
b. 1 and 2 only
1800m above the sea level and their
occurence decreases with the increase c. 1 and 3 only
in altitude.
d. 1, 2 and 3
9. a
10. a
3) Which of the following tribe(s) is/are not
 Operation Vijay (1961) refers to belonging to the state of Sikkim?
the annexation of Goa by the Indian
1. Bhutia
Government from the Portuguese forces.
2. Lepcha
 Commemorating the 20th anniversary
of 'Operation Vijay (1999)', defence 3. Limboo
minister Rajnath Singh has recently ignited a 4. Tamang
'victory flame‟ from the National War
Memorial. Select the correct answer using the codes given
below:
 Operation Vijay, wherein brave soldiers of the
Indian Army overcame seemingly a. 4 only
insurmountable odds, hostile terrain, b. 1 and 4 only
weather, and a determined enemy occupying
dominating heights, to win the Kargil war. c. 1, 2 and 4 only
d. None of the above

www.shankariasacademy.com | www.iasparliament.com
35

4) Consider the following statements with 3. Budget 2019-20 has proposed to levy
respect to Van Mitra Portal zero MDR for all merchants and as well
as Customers.
1. It is an online portal launched by the
Union Ministry of Tribal Affairs. Which of the statement(s) given above is/are
correct?
2. It acts as a platform for citizens to
report poaching and illegal trade of a. 3 only
forest resources.
b. 1 and 2 only
Which of the statement(s) given above is/are
c. 2 and 3 only
correct?
d. 1, 2 and 3
a. 1 only
8) How is the Polar Satellite Launch
b. 2 only
Vehicle (PSLV) different from
c. Both 1 and 2 the Geosynchronous Satellite Launch
Vehicle(GSLV)?
d. Neither 1 nor 2
1. PSLV is designed mainly to deliver the
earth-observation or remote-sensing
5) FinSpy sometimes seen in the news recently satellites, whereas the GSLV is designed
is a mainly to deliver the communication-
satellites.
a. Bio capsule
2. PSLV does not have enough power to
b. Spyware virus carry heavier satellites, whereas GSLV
c. Vaccine for TB can carry heavier payloads.
d. Disinfectant resistant Pathogen 3. PSLV does not have enough power to go
deeper into space, whereas GSLV can
travel deeper into space.
6) With respect to Related Party Transactions Select the correct answer using the codes given
(RPT), consider the following statements below:
1. These are transactions that a company a. 2 only
does with parties related to it.
b. 1 and 2 only
2. In India, Related Party Transactions are
banned under the Companies Act, 2013. c. 2 and 3 only
Which of the statement(s) given above is/are d. 1, 2 and 3
correct?
a. 1 only 9) Which one of the following best describes
b. 2 only the term Trade Deficit?
c. Both 1 and 2 a. It occurs when the total expenditures exceed
the revenue that it generates
d. Neither 1 nor 2
b. It occurs when a country's exports exceed its
imports
7) With respect to Merchant Discount Rate c. It is a part of fiscal deficit indicating the
(MDR), consider the following statements: difference in interstate trade
1. It is the sum total of all the charges and d. None of the above
taxes that a digital payment entails.
2. It is alternatively referred to as the
Transaction Discount Rate (TDR).

www.shankariasacademy.com | www.iasparliament.com
36

10) Consider the following statements with and accessibility criteria must be met and
respect to El Nino maintained.
1. It is a condition wherein the surface  Japan and South Korea are the only
waters in the equatorial Pacific Ocean, countries in South and South-Eastern Asia to
off the coast of South America, become have Blue Flag beaches.
unusually warm.
 The Union Environment Ministry has recently
2. It is known to suppress rainfall in India selected 12 beaches in India to vie for a „Blue
during the monsoon months. Flag‟ certification.
Which of the statement(s) given above 3. d
is/are incorrect?
List of Tribes in Sikkim:
a. 1 only
1. Bhutia (including Chumbipa, Dopthapa,
b. 2 only Dukpa, Kagatey, Sherpa, Tibetan, Tromopa,
c. Both 1 and 2 Yolmo)

d. Neither 1 nor 2 2. Lepcha


3. Limboo
4. Tamang
Answers
 A proposal for reservation of seats for Limboo
1. b
and Tamang communities in Sikkim
 A team of researchers has recently discovered Legislative Assembly is under consideration of
a new specimen of a microraptor -- volant the Government of India.
dromaeosaurid Microraptor zhaoianus -- with 4. d
the remains of a nearly complete lizard
preserved in its stomach.  The Madhya Pradesh government has
recently began organising gram sabhas in
 The researchers have named the lizard villages to consider afresh rejected claims of
after Lord Indra. Scheduled Tribes and other traditional forest
 The lizard is unlike any previously known from dwellers to forest land rights under the Forest
the Cretaceous and represents a new species- Rights Act (FRA), 2006.
Indrasaurus wangi.  To streamline the application
 The name Indrasaurus was inspired by a Vedic process and deal with the bulk of
legend in which the god Indra was claims, the State has also launched an online
swallowed by a dragon during a great application system, Van Mitra, in the
battle (the dragon here referring to Hoshangabad district on a pilot basis.
Microraptor). 5. b
2. a  FinSpy is the latest version of spyware
 The iconic Blue Flag is one of the world‘s most virus designed to track nearly every
recognised voluntary eco-labels awarded interaction on a mobile device.
to beaches, marinas, and sustainable boating  FinSpy collects data from instant messengers—
tourism operators. even if they are using encryption programs—
 The Blue Flag Programme for beaches and such as Russian developed messaging app
marinas is run by the international, non- Telegram, Facebook‘s WhatsApp, Skype,
governmental, non-profit organisation FEE Signal, China‘s WeChat and BlackBerry
(the Foundation for Environmental Messenger.
Education).  It can work both on iOS and Android devices.
 In order to qualify for the Blue Flag, a series of 6. a
stringent environmental, educational, safety,

www.shankariasacademy.com | www.iasparliament.com
37

 Related Party Transactions are just that  PSLV is designed mainly to deliver the ―earth-
— transactions that a company does with observation‖ or ―remote-sensing‖ satellites
parties related to it. with lift-off mass of up to about 1750 Kg to
Sun-Synchronous circular polar orbits of 600-
 So, if Company A buys goods or services from 900 Km altitude.
its director X, it counts as an RPT.
 The GSLV is designed mainly to deliver the
 In RPTs, there is a risk that the related party communication-satellites to the highly
may be favoured with terms that could harm elliptical (typically 250 x 36000 Km)
the interests of the company‘s shareholders. Geosynchronous Transfer Orbit (GTO).
 For instance, if Company A rents office  PSLV does not have enough power to carry
premises from Y, a relative of its director X, heavier satellites, or to go deeper into space.
and pays higher than market rent in a
sweetheart deal, it helps Y and indirectly X, but  GSLV can carry heavier payloads and travel
harms shareholders of Company A. deeper into space.
 That said, RPTs need not always be bad for a 9. d
company‘s shareholders.
 A trade deficit is an economic measure of
 In many cases, RPTs make commercial and international trade in which a country's
operational sense for the company. imports exceed its exports.
 So, the Companies Act has not banned  A trade deficit represents an outflow of
RPTs but instead laid down safeguards to be domestic currency to foreign markets.
followed.
 It is also referred to as a negative balance of
 As a result, RPTs, in general, need the approval trade (BOT).
of the company‘s Board of Directors and the
 Trade Deficit = Total Value of Imports –
Audit Committee.
Total Value of Exports
7. d
10. d
 Merchant Discount Rate (alternatively referred
 El Nino is a condition wherein the surface
to as the Transaction Discount Rate or
waters in the equatorial Pacific Ocean, off
TDR) is the sum total of all the charges and
the coast of South America,
taxes that a digital payment entails.
become unusually warm.
 For instance, the MDR includes bank charges,
 Its opposite condition, when it becomes
which a bank charges customers and
unusually cold, is termed, La Nina.
merchants for allowing payments to be made
digitally.  The two conditions influence weather events
worldwide, including the Indian monsoon.
 Similarly, MDR also includes the processing
charges that a payments aggregator has to pay  El Nino is known to suppress
to online or mobile wallets or indeed to banks rainfall during the monsoon months.
for their service.
 Finance Minister, in the Union Budget 2019-
20 has proposed that no charges or Zero 17-07-2019
Merchant Discount Rate (MDR) will be 1) Which one of the following is the objective
imposed on customers as well as of Kan Sikul, Kan Huan Scheme which was
merchants. launched recently in the state of Mizoram?
 The payments industry has raised concerns a. To make schools self-sufficient in the local
over the Budget proposal to levy zero MDR on variety of fruits and vegetables by March 2020
all merchants and has said it will lead to the
collapse of the payments acquiring industry. b. To promote recycling of grey water by giving
equipments at subsidised rate
8. d

www.shankariasacademy.com | www.iasparliament.com
38

c. To encourage people to do away the practice of c. 2 and 3 only


shifting cultivation
d. 1, 2 and 3
d. To promote export of milk and reduce the
wastage by 50% by 2022
5) Consider the following statements
1. It is a serene marshland with shallow
2) With respect to National Programme for
waters located in the state of Gujarat.
Prevention and Control of Deafness (NPPCD),
consider the following statements 2. It is a wetland site designated to be of
international importance under the
1. It is a 100% Centrally Sponsored
Ramsar Convention.
Scheme.
3. It is a paradise for migratory birds
2. It aims to reduce hearing impairment
declared as a Ramsar site in 2012.
and deafness burden by 25% of the
existing burden by the end of 12th Five Identify the sanctuary that correctly matches
Year Plan. with the above description:
Which of the statement(s) given above is/are a. Marine Sanctuary
correct?
b. Nalsarovar Bird Sanctuary
a. 1 only
c. Great Rann Wildlife Sanctuary
b. 2 only
d. Chharidhund Conservation Reserve
c. Both 1 and 2
d. Neither 1 nor 2
6) Wolbachia sometimes seen in the news
recently is a
3) The State of Food Security and Nutrition in a. Naturally occurring bacteria
the World 2019 report was published recently
by which of the following organisations? b. Strain of Aedes aegypti mosquito

a. International Food Policy Research Institute c. World‘s first nuclear powered aircraft

b. UN Food and Agriculture Organization d. Festival celebrated in Nagaland

c. World Health Organisation


d. None of the above 7) Biplab Kumar Sarma Committee sometimes
seen in the news recently is associated with
which of the following?
4) Consider the following statements with a. Assam Accord
respect to Chandipura Vesiculovirus (CHPV)
b. Naga Peace Process
1. The virus spreads mainly through the
bite of sand flies. c. Instrument of Accession

2. It predominantly infects children aged d. Naxal insurgency


below 14.
3. It is known to cause inflammation of the 8) Consider the following statements with
brain, progress rapidly from an respect to Internet Saathi Programme
influenza-like illness to coma and death.
1. It aimed to empower rural women on
Which of the statement(s) given above is/are how to use the Internet.
correct?
2. It was launched by Microsoft along with
a. 3 only Tata Trusts.
b. 1 and 2 only

www.shankariasacademy.com | www.iasparliament.com
39

Which of the statement(s) given above is/are Lawngtlai self-sufficient in the local variety of
correct? fruits and vegetables by March 2020.
a. 1 only  This is to reduce malnutrition and stuntedness
among children.
b. 2 only
2. b
c. Both 1 and 2
d. Neither 1 nor 2  As per the guidelines of National Health
Mission (NHM), the funding pattern between
the Centre and State under the National
9) Consider the following pairs Programme for Prevention and Control of
Deafness (NPPCD) is 60:40 for all states
1. Neelum Valley – Jammu and Kashmir except for the North Eastern and Himalayan
States where the ratio is 90:10.
2. Spiti Valley – Himachal Pradesh
Which of the pair(s) given above is/are  In case of Union Territories, Central funding is
correctly matched? 100 % for all UTs except the UTs with
legislature (i.e. Delhi and Puducherry), where
a. 1 only this ratio is 60:40.
b. 2 only  Its long term objective is to prevent and
c. Both 1 and 2 control major causes of hearing impairment
and deafness, so as to reduce the total disease
d. Neither 1 nor 2 burden by 25% of the existing burden by the
end of 12th Five Year Plan.

10) Consider the following statements 3. b


regarding coral bleaching  The State of Food Security and Nutrition in the
1. It occurs when coral polyps expel World 2019 report, released by the UN Food
zooxanthellae that live inside their and Agriculture Organization(FAO),
tissues. estimated that 820 million people worldwide
did not have enough to eat in 2018, up from
2. Increase in sea surface temperature is 811 million in the previous year.
the only cause for coral bleaching.
 At the same time, the number of overweight
Which of the statements given above is/are individuals and obesity continue to increase in
correct? all regions.
a. 1 only 4. d
b. 2 only  The Chandipura Vesiculovirus (CHPV), first
c. Both 1 and 2 discovered by two virologists of the Pune-
based National Institute of Virology (NIV) in
d. Neither 1 nor 2 1965, predominantly infects children.
 The majority of infected patients are children
Answers aged below 14.

1. a  Named after the town in Maharashtra where


the discovery was made and isolated,
 Kan Sikul, Kan Huan. In the Mizo language, it Chandipura virus is known to cause
means ‗My School, My Farm‘. inflammation of the brain, progress rapidly
from an influenza-like illness to coma and
 It was recently launched in the Lawngtlai death.
district of Mizoram.
 The virus spreads mainly through the bite of
 The aim of the scheme is to make every school, sand flies, and sometimes through mosquitoes.
Anganwadi, child care institution and hostel in

www.shankariasacademy.com | www.iasparliament.com
40

5. b  Launched as a pilot project in 2015, along with


Tata Trusts, 'Internet Saathi' programme
 Nalsarovar is a serene marshland with
focuses on educating women on how to use the
shallow waters that is spread over 120 sq km,
Internet.
about 60 km away from Ahmedabad.
9. c
 A paradise for migratory birds, Nalsarovar was
declared a Ramsar site in 2012. 10. a
 It is a wetland site designated to be of
international importance under the Ramsar
Convention.
18-07-2019
1) Ploonet sometimes seen in the news recently
 Seven islands that are a part of the protected
refers to which of the following?
Nalsarovar Bird Sanctuary located near
Ahmedabad are facing human encroachment. a. These are moons that escape their planet and
behave like a planet
6. a
b. Amazon‘s project to connect rural areas with
 To control the spread of dengue and low cost internet connectivity
chikungunya, scientists at the ICMR have
developed a strain of the Aedes aegypti c. A type of technical textile used in the health
mosquito — the main vector that transmits the care sector
viruses that cause dengue — into
d. It is a project aimed at finding reasons for the
which Wolbachia, a naturally occurring
dwarfness of the planet Pluto
bacteria, is introduced.
 Wolbachia inhibits viral infection, which
means, people will feel the mosquito bite but 2) Which of the following is/are the
they will not be infected. applications of IRNSS?
 The bacteria will not allow the virus to 1. Navigational purposes
replicate in the mosquito and so, the virus 2. Disaster management
won‘t be transmitted through a bite.
3. Geodetic data capture
7. a
4. Fleet management
 The Government has notified a high-level
committee that is expected to look into the Select the correct answer using the codes given
implementation of a clause of Assam below:
Accord that seeks to provide safeguards to a. 1 only
preserve and promote social, cultural and
linguistic identity of Assamese people. b. 1 and 4 only
 The committee, headed by retired Gauhati c. 1, 2 and 4 only
High Court judge Biplab Kumar Sarma, d. 1, 2, 3 and 4
will also make recommendations on
reservation for Assamese people in the
Assembly, and other local bodies. 3) Consider the following statements with
8. a respect to Ebola

 Google India's 'Internet Saathi' programme 1. The virus is most often spread by close
aimed to empower rural women on how to use contact with bodily fluids of people.
the Internet has now added two more states -- 2. People living with HIV, malnutrition or
Punjab and Odisha. diabetes, or people who use tobacco,
 With this, the programme has now reached 2.6 have a much higher risk of falling ill.
lakh villages in 20 states.

www.shankariasacademy.com | www.iasparliament.com
41

3. WHO has recently declared the Ebola 3. Judgments delivered by the Court in
crisis in the DR Congo a public health disputes between States are binding
emergency of International concern. upon the parties concerned.
Which of the statement(s) given above is/are Which of the statement(s) given above is/are
correct? correct?
a. 1 only a. 1 only
b. 1 and 2 only b. 1 and 2 only
c. 1 and 3 only c. 2 and 3 only
d. 1, 2 and 3 d. 1, 2 and 3

4) “Find the Incredible You” is a campaign 7) With respect to Atal Bimit Vyakti Kalyan
launched by which of the following ministries? Yojana, consider the following statements:
a. Ministry of Tourism 1. Under the Scheme, the government will
pay 8.33% EPS contribution of the
b. Ministry of Sports and Youth Welfare
employer for the new employment.
c. Ministry of Social Justice and Empowerment
2. All industries registered with
d. Ministry of Human Resources and Employees' Provident Fund
Development Organization (EPFO) can apply for
availing benefits under the scheme.
Which of the statement(s) given above is/are
5) With respect to National Investigation correct?
Agency (Amendment) Bill, 2019, consider the
following statements: a. 1 only
1. It enhances the jurisdiction of NIA to b. 2 only
investigate scheduled offences
c. Both 1 and 2
committed outside India targeting
Indians as well as Indian assets abroad. d. Neither 1 nor 2
2. It curtails the functions of NIA to
investigate offences related to human
8) Which of the following are not the
trafficking and counterfeit currency.
components of Indian Cyber Crime
Which of the statement(s) given above is/are Coordination Centre (I4C) Scheme?
correct?
1. National Cybercrime Training Centre
a. 1 only
2. National Cybercrime Reporting Portal
b. 2 only
3. National Cyber Research and
c. Both 1 and 2 Innovation Centre
d. Neither 1 nor 2 4. National Cybercrime Forensic
Laboratory Ecosystem
Select the correct answer using the codes given
6) With respect to International Court of
below:
Justice (ICJ), consider the following
statements: a. None
1. It is the principal judicial organ of the b. 1 and 3 only
United Nations.
c. 2 and 3 only
2. Only States are eligible to appear before
d. 1, 2, 3 and 4
the Court in contentious cases.

www.shankariasacademy.com | www.iasparliament.com
42

 The IRNSS can provide Standard Positioning


Service (SPS) to all users, and an encrypted
9) Consider the following statements with
Restricted Service (RS) to authorised users.
respect to Long Period Average (LPA)
1. It is the average rainfall received by the  The IRNSS has a position accuracy better than
country as a whole during the south- 20 metres in the primary service area.
west monsoon, for a 100-year period. It can help in
2. This acts as a benchmark against which 1. Terrestrial, aerial and marine navigation
the rainfall in any monsoon season is
measured. 2. Disaster management

Which of the statement(s) given above is/are 3. Vehicle tracking and fleet management
correct? 4. Integration with mobile phones
a. 1 only 5. Precise timing
b. 2 only 6. Mapping and Geodetic data capture
c. Both 1 and 2 7. Terrestrial navigation aid for hikers and
d. Neither 1 nor 2 travellers
8. Visual and voice navigation for drivers

10) Consider the following statements with 3. c


respect to Contract Farming  Ebola is a virus that initially causes sudden
1. It can be defined as an agricultural fever, intense weakness, muscle pain and a
production carried out according to an sore throat.
agreement between a buyer and  It progresses to vomiting, diarrhoea and both
farmers. internal and external bleeding.
2. The buyer has the mandatory obligation
 People are infected when they have direct
to provide the supply of farm inputs,
contact through broken skin, or the mouth and
land preparation and the provision of
nose, with the blood, vomit, faeces or bodily
technical advice.
fluids of someone with Ebola.
Which of the statement(s) given above is/are
 Patients tend to die from dehydration and
correct?
multiple organ failure.
a. 1 only
 Health care workers are often at high risk.
b. 2 only
 The World Health Organization has recently
c. Both 1 and 2 declared the Ebola crisis in the Democratic
d. Neither 1 nor 2 Republic of Congo a "public health
emergency of international concern".
 For such a declaration, an outbreak must
Answers constitute a risk to other countries and
require a coordinated response.
1. a
 Declaring a disease as global health
 If a moon of an exoplanet turns rogue (as it emergency often brings an increase in
moves nearer to its star, breaking away —
international attention and aid.
or being forced out of its orbit by the planet)
and going off on its own trip, in effect behaving 4. a
like a planet in its own right is called as a
―Ploonet‖.  The "Find the Incredible You" campaign of
the Tourism Ministry, has won the Pacific
2. d

www.shankariasacademy.com | www.iasparliament.com
43

Asia Travel Association (PATA) Gold  The scheme covers Insured Persons (IP)
Award, 2019. covered under the Employees‗ State Insurance
Act, 1948.
 The campaign focuses on digital and social
media and the promotion of niche tourism  It aims to provide cash relief to
products of the country. unemployed insured person.
5. a  The relief will be payable in cash directly to
their Bank Account in case of unemployment
 Parliament has recently passed the National
and while they search for new engagement.
Investigation Agency (Amendment) Bill, 2019.
8. a
 The legislation aims at enhancing the
jurisdiction of NIA to investigate scheduled  Ministry of Home Affairs (MHA) has rolled out
offences committed outside a scheme ‗Indian Cyber Crime
Indiatargeting Indians as well as Indian Coordination Centre (I4C)‘ for the period
assets abroad. 2018-2020, to combat cyber crime in the
country, in a coordinated and effective
 The latest amendment will enable the NIA
manner.
to additionally investigate offences
related to human trafficking, counterfeit  The scheme has following seven
currency, manufacture or sale of prohibited components:
arms, cyber-terrorism and offences under the
Explosive Substances Act, 1908. 1. National Cybercrime Threat Analytics Unit.
2. National Cybercrime Reporting Portal.
 It also provides for constituting special
courts for the trial of scheduled offences such 3. Platform for Joint Cybercrime Investigation
as human trafficking and cyber terrorism. Team.
6. d 4. National Cybercrime Forensic Laboratory
Ecosystem.
 The Court is the principal judicial organ of
the United Nations. 5. National Cybercrime Training Centre.
 It was established by the United Nations 6. Cybercrime Ecosystem Management Unit.
Charter, which was signed in 1945 in San 7. National Cyber Research and Innovation
Francisco (United States), and began work in Centre.
1946 in the Peace Palace, The Hague
(Netherlands). 9. b
 Only States are eligible to appear before  LPA is the average rainfall received by the
the Court in contentious cases. country as a whole during the south-west
monsoon, for a 50-year period.
 The Court has no jurisdiction to deal with
applications from individuals, non-  The current LPA is 89 cm, based on the
governmental organizations, average rainfall over years 1960 and 2010.
corporations or any other private entity.
 This acts as a benchmark against which the
 Judgments delivered by the Court (or by one of rainfall in any monsoon season is measured.
its Chambers) in disputes between States
are binding upon the parties concerned.  The India Meteorological Department (IMD)
is likely to revise downward by 2% the
7. d quantity of „normal‟ monsoonal rain.
 Employees' State Insurance 10. a
Corporation under Ministry of Labour and
Employment has rolled out Atal Bimit Vyakti  The buyer has no obligation to provide the
Kalyan Yojna. supply of farm inputs, land preparation and
the provision of technical advice. (It is
voluntary only)

www.shankariasacademy.com | www.iasparliament.com
44

19-07-2019 4) Which of the following rivers flows through


the Kaziranga National Park?
1) Which of the following is the application
of Resveratrol, commonly found in grape skin 1. Payaswini
and blueberries? 2. Rangpo
a. It acts as a sedative agent widely used in health 3. Diphlu
care sector
4. Mora Dhansiri
b. To keep astronauts preserve their muscle mass
during trips in space 5. Brahmaputra
c. To act as a preservative in personal care Select the correct answers using the codes
products given below:
d. To help mountaineers preserving their body a. 3 and 5 only
against low atmospheric pressure b. 3, 4 and 5 only
c. 2, 3, 4 and 5 only
2) Consider the following statements d. 1, 2, 3, 4 and 5
1. It is located near the equator in the
Pacific Ocean.
5) Consider the following statements with
2. It is slightly west of the International respect to Hand Made in
Date Line. India project sometimes seen in the news
3. Geographically, it is part of the larger recently
island group of Micronesia. 1. It aimed at capacity building for 5000
Identify the Island based on the description weavers, artisans and traders to
given above: promote hand-made art in five states.
a. Tahiti Island 2. It was launched by the Ministry of skill
development and Entrepreneurship.
b. Hawaii Island
Which of the statement(s) given above is/are
c. Bouvet Island correct?
d. Marshall Islands a. 1 only
b. 2 only
3) Consider the following statements with c. Both 1 and 2
respect to IUCN Red List
d. Neither 1 nor 2
1. Species added to IUCN Red List has
been increased at an unprecedented
rate in recent times. 6) Larak Island sometimes seen in the news
2. Habitat destruction and hunting by recently is located in which of the following
humans are the major reasons for region?
decrease in the number of species. a. Persian Gulf
Which of the statement(s) given above is/are b. Bay of Bengal
correct?
c. Andaman Sea
a. 1 only
d. Mediterranean Sea
b. 2 only
c. Both 1 and 2
d. Neither 1 nor 2

www.shankariasacademy.com | www.iasparliament.com
45

7) Consider the following pairs b. 2 only


1. Narasimham Committee – Banking c. Both 1 and 2
reforms
d. Neither 1 nor 2
2. Aruna Sundarajan Committee –
Telecom Sector
10) The term “INS Kalvari”, sometimes seen in
3. Bimal Jalan Committee – RBI reforms
the News, is related to
Which of the pair(s) given above is/are
a. Nuclear powered submarine
correctly matched?
b. Indigenous amphibious warship
a. 3 only
c. Scorpene class submarine
b. 1 and 2 only
d. Stealth guided missile destroyer
c. 1 and 3 only
d. 1, 2 and 3
Answers
8) Consider the following statements with 1. b
respect to Serious Fraud Investigation Office  A recent study has showed that the red wine
(SFIO) component ―Resveratrol‖ commonly
1. It is a multi-disciplinary organization found in grape skin and
under Ministry of Corporate Affairs. blueberries substantially preserves muscle
mass and strength in rats exposed to the
2. It was established in January 2003 wasting effects of simulated Mars gravity.
based on the recommendation of
Naresh Chandra Committee on 2. d
corporate governance. 3. c
Which of the statement(s) given above is/are  Mankind‘s destruction of nature is driving
correct? species to the brink of extinction at an
a. 1 only unprecedented rate, the IUCN warned
recently, as it added more than 7000 animals,
b. 2 only fish and plants to its endangered ―Red List‖.
c. Both 1 and 2  Habitat destruction and hunting by humans
d. Neither 1 nor 2 are reasons for such fall in numbers.
4. b
9) Consider the following statements with  The park area is circumscribed by
regard to “Recognition of National Party” by the Brahmaputra River, which forms the
the Election Commission of India northern and eastern boundaries, and
the Mora Diphlu, which forms the southern
1. A party which got recognition as a state
boundary.
party in at least four states would be
recognized as a National Party  Other notable rivers within the park are
the Diphlu and Mora Dhansiri.
2. For granting recognition as a national
party, the party‟s performance over two  Rangpo – Sikkim
consecutive Lok Sabha or assembly
elections will be considered  Payaswini – Kerala

Which of the statements given above is/are 5. a


correct?  Entrepreneurship Development
a. 1 only Institute of India (EDII) (an autonomous

www.shankariasacademy.com | www.iasparliament.com
46

and not-for-profit institute) has developed


a plan named ‗Hand Made in India‘ (HMI).
2) Which of the following is/are the reasons for
 It will be implemented in three years and in Assam being a flood-prone region?
the first phase; it has been extended to six
1. Deforestation
centres.
2. Incessant Rainfall
 It aimed to develop entrepreneurial
competencies in about 5,000 weavers, artisans 3. River Brahmaputra
and traders to promote hand-made art. 4. Earthquake prone Region
6. a 5. Population growth in catchment areas
 Iran's Revolutionary Guard Corps (IRGC) Select the correct answer using the codes given
stopped and seized an oil tanker near Larak below:
Island in the Strait of Hormuz.
a. 2 and 3 only
7. d
b. 2, 3 and 5 only
1. Narasimham Committee – Banking reforms
c. 1, 2, 3 and 5 only
2. Aruna Sundarajan Committee – Telecom
Sector d. 1, 2, 3, 4 and 5
3. Bimal Jalan Committee – RBI reforms
8. c 3) Deeksharambh – a guide to Student
Induction Programme was recently released by
 SFIO was established in January 2003 based which of the following ministries?
on the recommendation of Naresh Chandra
Committee on corporate governance. a. Ministry of Home affairs
9. c b. Ministry of Women and Child Development
10. c c. Ministry of Social Justice and Empowerment
d. Ministry of Human Resources and
Development
20-07-2019
1) Consider the following statements with
respect to Paramarsh Scheme which was 4) Consider the following statements with
launched recently respect to Unified District Information on
School Education (UDISE)Report, 2017-
1. Under the scheme, the leading 18 which was recently tabled in the Parliament
institutions will provide regular
mentoring to help colleges achieve high 1. Tripura, Sikkim, and Nagaland have the
quality standards. best Pupil-Teacher Ratio (PTR) in India.

2. It will be operationalized through a 2. Among larger states, Kerala has the best
“Hub & Spoke” model. Right to Education (RTE) compliance.

3. It was launched by the Ministry of 3. Larger states showed a better Pupil-


Human Resources Development. Teacher Ratio (PTR) than smaller states
and Union Territories.
Which of the statement(s) given above is/are
correct? Which of the statement(s) given above is/are
correct?
a. 3 only
a. 2 only
b. 1 and 3 only
b. 1 and 2 only
c. 2 and 3 only
c. 2 and 3 only
d. 1, 2 and 3

www.shankariasacademy.com | www.iasparliament.com
47

d. 1, 2 and 3 2. The prime objective of the mission is to


conduct deep ocean hydrographic
survey in the Antarctic Region.
5) Consider the following statements with
Which of the statement(s) given above is/are
respect to North Eastern Region Community
correct?
Resource Management Project (NERCORMP)
a. 1 only
1. It is a World Bank aided project
implemented by Ministry b. 2 only
for Development of North Eastern
c. Both 1 and 2
Region.
d. Neither 1 nor 2
2. It aims for the holistic development of
north east region and is operational in
all north eastern states.
8) Consider the following statements with
3. It provides skill development training, respect to North East Rural Livelihood Project
vocational training to unemployed (NERLP)
youths & Self Help Groups (SHGs)
members for job placement and self- 1. It aims to promote enterprise
employment. development by establishing community
based micro-credit organizations and
Which of the statement(s) given above is/are non-farm enterprises in its project
correct? areas.
a. None 2. It is being operational only in the states
of Mizoram, Nagaland, Sikkim and
b. 1 only
Tripura.
c. 1 and 3 only
3. It is being implemented by the Ministry
d. 1, 2 and 3 of Development of North Eastern
Region.
Which of the statement(s) given above is/are
6) Consider the following statements with
correct?
respect to Solar Charkha Mission
a. 1 only
1. It was launched for the implementation
of 50 Solar Charkha Clusters across the b. 1 and 3 only
country to generate direct employment
c. 2 and 3 only
for nearly one lakh persons.
d. 1, 2 and 3
2. It is being implemented by the Ministry
of New and Renewable Energy.
Which of the statement(s) given above is/are 9) INS Sagardhwani sometimes seen in the
correct? news recently is a
a. 1 only a. Offshore Patrol Vessel of Indian Navy
b. 2 only b. Oceanographic research vessel of DRDO
c. Both 1 and 2 c. Deep submergence Rescue Vehicle of Indian
Coast Guard
d. Neither 1 nor 2
d. Hydrographic Survey vessel of Indian Navy

7) Consider the following statements with


respect to Mission Sagar Maitri – 2 10) Which of the following countries is/are
members of G7 Grouping?
1. It is an initiative of Defence Research
and Development Organisation (DRDO). 1. India

www.shankariasacademy.com | www.iasparliament.com
48

2. France  Following the devastating earthquake of 1950,


the level of the Brahmaputra rose by two
3. Canada
metres in Dibrugarh area in eastern Assam.
4. China
 Besides these natural factors are the man-
Select the correct answer using the codes given made ones — habitation, deforestation,
below population growth in catchment
a. All except 1 areas(including in China) — which lead to
higher sedimentation.
b. All except 4
 For example, the sediment deposition itself
c. All except 1 and 4 creates temporary sandbars or river islands.
d. All of the above 3. d
 Ministry of Human Resource
Development has recently released the
Answers ―Deeksharambh‖, an UGC Guide to Student
1. d Induction Programme.
 MHRD has recently  Deeksharambh aims to help new students
launched „Paramarsh‘ – a University adjust and feel comfortable in the new
Grants Commission (UGC) scheme environment, inculcate in them the ethos and
for Mentoring National Accreditation culture of the institution, help them build
and Assessment Council (NAAC) bonds with other students and faculty
Accreditation Aspirant Institutions to members, and expose them to a sense of larger
promote Quality Assurance in Higher purpose and self-exploration.
Education.
4. b
 The scheme will be a paradigm shift in the
 Tripura, Sikkim, and Nagaland have the best
concept of mentoring of institution by another
Pupil-Teacher Ratio (PTR), according to a
well performing institution to upgrade their
report compiled by the Unified District
academic performance and enable them to get
Information on School Education (UDISE) in
accredited.
2017-18 and tabled by the government in
 The Scheme will be operationalized through a Parliament.
―Hub & Spoke‖ model wherein the Mentor
 Smaller states and UTs showed a better
Institution, called the ―Hub‖ is centralized and
PTR than larger states.
will have the responsibility of guiding the
Mentee institution through the secondary  The PTR norm in the country has been set at
branches the ―Spoke‖ through the services 30:1 for primary level, and at 35:1 for the
provided to the mentee for self-improvement. upper primary level.
2. d  The report found the PTR at national level 23:1
for primary schools and 24:1 for upper primary
 Apart from incessant rainfall during the
schools.
monsoon, there are many contributory
factors that causes floods in Assam, natural  Bihar, with 67.94% primary schools and
and man-made. 77.86% upper primary schools non-compliant
with the standards, as well as a PTR of 38:1
 At the crux is the very nature of the river
and 39:1 in those schools respectively, is at
Brahmaputra —dynamic and unstable.
the bottom on all four counts.
 Because of the earthquake-prone nature
 UP and Jharkhand too have poor ratios and
of the region, the river Brahmaputra has not
compliance ratios.
been able to acquire a stable character.
 Among larger states, Kerala has the best
RTE compliance, with less than 5% non-

www.shankariasacademy.com | www.iasparliament.com
49

compliance at both primary and upper primary among Indian Ocean Rim (IOR)
schools. countries.
5. a  INS Sagardhwani, a Marine Acoustic
Research Ship (MARS) of the Naval Physical &
 North Eastern Region Community Resource
Oceanographic Laboratory (NPOL), Kochi
Management Project (NERCORMP) is a
and operated by the Indian Navy has set
livelihood and rural development project
sail from Kochi for a scientific venture called
aimed to transform the lives of the poor and
‗Marine & Allied Interdisciplinary
marginalized tribal families in North East (NE)
Training and Research Initiative
India.
(MAITRI)‘.
 NERCORMP is a joint developmental intiative
 The prime objectives of the Mission are data
of the North Eastern
collection from the entire North Indian
Council (NEC), Ministry of
Ocean, focussing on the Andaman Sea and
DoNER and International Fund for
adjoining seas and establishing long-term
Agricultural Development (IFAD).
collaboration with eight IOR countries in the
 Objective: To improve the livelihoods of field of ocean research and development.
vulnerable groups in a sustainable manner
 The Mission commemorates the Golden
through improved management of their
Jubilee Celebrations of India‘s lone research
resource base in a way that contributes to
ship INS Kistna‘s missions as part of the
the preservation and restoration of the
historic International Indian Ocean
environment.
Expeditions (IIOE), which took place during
 The scheme has facilitated in establishing 1962-65.
community based micro-credit
 As part of the mission, INS Sagardhwani
organizations and non-farm
will revisit the selected tracks of INS
enterprises in its project areas which
Kistna and provide NPOL scientists ample
provides employment to Self Help Groups
opportunities to collaborate and garner a close
(SHGs).
working relationship with the oceanographic
 It is operational in 10 districts of 4 States of counterparts of the IOR countries.
NER namely Arunachal Pradesh, Assam, 8. c
Manipur and Meghalaya.
 North East Rural Livelihood Project (NERLP),
6. a
a World Bank aided project, being
 The Ministry of MSME has launched the implemented by Ministry of
Mission Solar Charkha for implementation of DoNER, provides skill development
50 Solar Charkha Clusters across the country. training, vocational training to
unemployed youths & Self Help Groups
 The scheme is envisaged to generate direct (SHGs) members for job placement and self-
employment for nearly one lakh persons. employment in 11 districts of four North East
 Solar Charkha units have been classified as States namely Mizoram, Nagaland,
Village Industries. Sikkim and Tripura.

 The scheme is being implemented by 9. b


Khadi and Village Industries  INS Sagardhwani is the Oceanographic
Commission (KVIC), Mumbai. Research Vessel (ORV) of Defence Research
7. a and Development Organisation
(DRDO), maintained and operated by
 It is a unique initiative of DRDO which aligns the Indian Navy.
with the broad objective of ―Safety And
Growth for All in the Region (SAGAR)‖  It is a ‗Marine Acoustic Research Ship‘ (MARS)
to promote closer co-operation in socio- designed and developed by ‗Naval Physical and
economic aspects as well as greater scientific Oceanographic Laboratory‘ (NPOL), Kochi.
interaction especially in ocean research

www.shankariasacademy.com | www.iasparliament.com
50

 The ship is fitted with state-of-the-art 2. It is being implemented under the


equipments like the latest wave height Ministry of Agriculture and Farmers
measuring radars, marine radio etc. welfare.
10. c Which of the following statement(s) given
above is/are correct?
Members of G7 grouping:
a. 1 only
 France, Germany, Italy, Japan, the United
States, the United Kingdom, and Canada. b. 2 only
c. Both 1 and 2

22-07-2019 d. Neither 1 nor 2

1) Consider the following statements with


respect to Ayush Grid Project 4) Export of honey and other bee hive products
1. It is the proposed IT backbone for the is being regulated in India by which of the
entire AYUSH sector in the fields of following agencies?
healthcare delivery at all levels, a. Export Inspection Council
research, education, schemes and
various health programs. b. Export Promotion Council of India

2. It will be developed jointly by the c. Food Safety and Standards Authority of India
Ministry of AYUSH and Ministry of d. Agricultural and Processed Food Products
Electronics and Information Export Development Authority
Technology.
Which of the statement(s) given above is/are
correct? 5) Consider the following statements with
respect to Safdarjung Tomb
a. 1 only
1. It is the last enclosed garden tomb in
b. 2 only Delhi in the tradition of Humayun's
c. Both 1 and 2 Tomb.

d. Neither 1 nor 2 2. It was built in 1753- 54 as mausoleum of


Safdarjung, the viceroy of Awadh.
Which of the statement(s) given above is/are
2) The main objective of Har Medh Par correct?
Ped campaign is?
a. 1 only
a. To promote agro-forestry
b. 2 only
b. To promote the cultivation of bio-fuel crops
c. Both 1 and 2
c. To promote the export of agricultural products
d. Neither 1 nor 2
d. To eliminate the use of fertilizers and
pesticides
6) Consider the following statements with
respect to Exercise Garuda 2019
3) Consider the following statements with
respect to Jaivik kheti portal 1. It is a bilateral Indo- Russia large force
employment warfare exercise.
1. It is a one stop solution for facilitating
organic farmers to sell their organic 2. It was hosted by Indian Air Force (IAF)
product and promoting organic farming at Noida, India.
and its benefits. Which of the statement(s) given above is/are
correct?

www.shankariasacademy.com | www.iasparliament.com
51

a. 1 only Which of the statements given above


is/are not correct?
b. 2 only
a. 1 only
c. Both 1 and 2
b. 2 and 3 only
d. Neither 1 nor 2
c. 1 and 2 only
d. 1, 2 and 3
7) Consider the following statements with
respect to Thirty Meter Telescope
1. Thirty Metre refers to the 30-metre 10) With reference to the concept of Vertical
diameter of the mirror which makes it Farming, which of the following will be the
three times as wide as the world‟s advantages of this concept?
largest existing visible-light telescope.
1. Under this farming the crops will be
2. It is being built by the Government of safe from extreme weather occurrences
United States of America in its territory such as droughts and floods.
of Hawaii.
2. This concept will reduce the use of
Which of the statement(s) given above is/are traditional fertilizers as well as
correct? chemical pesticides.
a. 1 only 3. In vertical faming more crops can be
grown in a small land area.
b. 2 only
4. Vertical farms use more water by
c. Both 1 and 2
depending on hydroponics or
d. Neither 1 nor 2 aeroponics.
Select the correct answer using the codes given
below
8) Global Innovation Index will be released
annually by which of the following a. 1 and 2 only
organizations?
b. 1, 2 and 3 only
a. World Intellectual Property Organisation
c. 2, 3 and 4 only
b. World Economic Forum
d. 1, 2, 3 and 4
c. UN Conference on Trade and Development
d. NITI Aayog
Answers
1. c
9) Consider the following statements with
respect to the Rajya Sabha  The AYUSH Grid Project is the proposed IT
backbone for the entire AYUSH
1. The states of India have unequal sector covering the healthcare systems
representation in the Rajya Sabha. Ayurveda, Yoga & Naturopathy, Unani,
2. All the seven Union Territories of India Siddha, Sowa Rigpa and Homoeopathy.
have equal representation in the Rajya  It is envisaged as an omnibus digital eco-
Sabha. system that would lead to all round
3. The President can nominate two development of the AYUSH sector in fields of
members from Anglo-Indian healthcare delivery at all levels, research,
Community if the community is not education, schemes and various health
adequately represented in the Rajya programs.
Sabha.  A Memorandum of Understanding (MoU) was
recently signed between Union Ministry of

www.shankariasacademy.com | www.iasparliament.com
52

Electronics and Information 7. a


Technology (MeitY) and Union Ministry
of AYUSH.  The telescope is being built by an
international
 As per MoU, MeitY has agreed to advice and collaboration of government
give technical support to AYUSH Ministry for organizations and educational institutions,
planning and development of AYUSH GRID at a cost of $1.4 billion.
Project.
 ―Thirty Metre‖ refers to the 30-metre diameter
2. a of the mirror, with 492 segments of glass
pieced together, which makes it three times as
 Under the ―Har Medh Par Ped‖
wide as the world‘s largest existing visible-light
campaign, agro forestry is being
telescope.
promoted for supplementing farm income,
increase risk management and climate  The larger the mirror, the more light a
resilient agriculture as an important telescope can collect, which means, in turn,
component of Integrated Farming Systems. that it can ―see‖ farther, fainter objects.
3. c 8. a
 Jaivik Kheti portal has been created for  GII will be published by World Intellectual
promotion and sale of organic produce to Property Organisation (WIPO) in
connect farmers involved in organic farming association with INSEAD and Cornell
with consumers directly for better prices. University.
 Already 80,000 farmers are registered on the 9. b
portal.
10. b
4. a
 The export of honey and other bee hive
products is being regulated by the Export 23-07-2019
Inspection Council (EIC), Ministry of 1) Project Sampark sometimes seen in the
Commerce & Industry. news recently is associated with which of the
5. c following?

 Safdarjung's Tomb is the last enclosed a. National Clean Air Programme


garden tomb in Delhi in the tradition of b. Border Roads Organisation
Humayun's Tomb, though it if far less grand in
scale. c. Defence Research and Development
Organisation
 It was built in 1753- 54 as mausoleum of
d. None of the above
Safdarjung, the viceroy of Awadh under
the Mughal Emperor, Mohammed Shah.
 It has several smaller pavilions with evocative 2) Consider the following statements with
names like Jangli Mahal, (Palace in the respect to Electronically Transmitted Postal
woods), Moti Mahal (Pearl Palace) Ballot System (ETPBS)
andBadshah Pasand (King's favourite).
1. It helps service voters to cast their vote
 The complex also has a madarsa. The on an electronically received postal
archaeological Survey of India maintains a ballot, from anywhere outside their
library over the main gateway. constituency.
6. d 2. It is a fully secured system, having two
security layers.
 Ex Garuda 2019, a bilateral Indo-
French large force employment warfare 3. There will be no duplication of casted
exercise hosted by French Air Force ETPB due to the unique QR Code in the
(FAF) at Mont-de-Marsan France. portal.

www.shankariasacademy.com | www.iasparliament.com
53

Which of the statements given above are 2. The vehicle has two solid strap-ons, a
correct? core liquid booster and a cryogenic
upper stage.
a. 1 and 2 only
3. The vehicle is designed to carry 10 tons
b. 1 and 3 only
of satellites into Geosynchronous
c. 2 and 3 only Transfer Orbit (GTO) or about 4 tons to
Low Earth Orbit (LEO).
d. 1, 2 and 3
Which of the statement(s) given above is/are
correct?
3) Urja Ganga Project, sometimes seen in the a. 2 only
news is related to
b. 1 and 2 only
a. Gas pipeline project aims to provide PNG and
CNG to residents and vehicles respectively c. 2 and 3 only
b. Massive solar power project over the canals of d. 1, 2 and 3
Ganga which aims to generate solar energy
c. It‘s a kind of ―Waste to Energy‖ process which
6) Consider the following statements with
will be implemented as a part of clean Ganga
respect to Chandrayaan-2
mission
1. It comprises a fully indigenous Orbiter,
d. It‘s a kind of hydroelectric power project aims
Lander and a Rover.
to generate 600 MW of electricity from Ganga
water 2. Its primary goal is to understand the
evolution of the Martian system and the
distribution of water molecules on its
4) A person can enroll him/herself as a Service surface.
Voter, if he/she is
Which of the statement(s) given above is/are
1. In the Armed Forces of India correct?
2. From the Central Armed Police Forces a. 1 only
(CAPF) of India
b. 2 only
3. Employed under the Government of
c. Both 1 and 2
India, in a post outside India
d. Neither 1 nor 2
4. Member of an Armed Police Force of a
State, and serving outside that state
Select the correct answer using the codes given 7) JATAN sometimes seen in the news recently
below: is a
a. 3 only a. Jel like Tablets
b. 2 and 3 only b. Malicious Software
c. 1, 2 and 3 only c. Fifth generation bio-fuel
d. 1, 2, 3 and 4 d. Virtual Museum Software

5) Consider the following statements with 8) Consider the following statements with
respect to GSLV Mk III respect to Protection of Human Rights
(Amendment) Bill, 2019
1. It is a two-stage launch vehicle
developed by ISRO. 1. It provides for an increase in the tenure
of Chairpersons of NHRC and SHRC to
five years from the current three years.

www.shankariasacademy.com | www.iasparliament.com
54

2. Under the bill, a former Supreme Court  Project Sampark was raised by Border
Judge can also become the NHRC Roads Organisation (BRO) in 1975 with its
Chairperson apart from the former Headquarters at Jammu.
Chief Justice of India (CJI).
2. d
3. Under the bill, the Members and
Chairperson shall be eligible for  Electronically Transmitted Postal Ballot
reappointment. System (ETPBS) is a fully secured system,
having two security layers.
Which of the statement(s) given above is/are
correct?  Secrecy of voting is maintained through the
use of OTP and PIN and no duplication of
a. 2 only casted ETPB is possible due to the unique QR
b. 1 and 2 only Code in the portal.

c. 2 and 3 only  Through this system the service voters cast


their vote on an electronically received postal
d. 1, 2 and 3 ballot, from anywhere outside their
constituency, thus reducing the chances of
losing the voting opportunity.
9) Hope Island has been developed as a world
Class Coastal & Eco Tourism Circuit in India. It 3. a
is located in which of the following states? 4. d
a. Kerala A person can enroll as a Service Voter if he/she is
b. Tamil Nadu 1. in the Armed Forces of India
c. Andhra Pradesh 2. from Assam Rifles, CRPF, BSF, ITBF; GREF in
d. Karnataka Border Road Organisation; Central Industrial
Security Force
3. employed under the Government of India, in a
10) Ministry of Tourism under the Swadesh post outside India
Darshan Scheme has recently identified
Tirthankar Circuit. The sites covered under 4. member of an Armed Police Force of a State,
this circuit are associated with which of the and serving outside that state
following? 5. a
a. Zoroastrianism  GSLV Mk III is a three-stage launch vehicle
b. Buddhism developed by ISRO.
c. Jainism  The vehicle has two solid strap-ons, a core
liquid booster and a cryogenic upper
d. None of the above stage.
 The vehicle is designed to carry 4 ton class of
Answers satellites into Geosynchronous Transfer
Orbit (GTO) or about 10 tons to Low
1. b Earth Orbit (LEO).
 Union Defence Minister has recently 6. a
inaugurated the One km long Ujh bridge in
Kathua district and 617.40 Metre long  Chandrayaan-2 is India's second mission to
Basantar bridge in Samba district of Jammu the moon.
& Kashmir.  It comprises a fully indigenous Orbiter,
 Both these bridges have been constructed Lander (Vikram) and Rover (Pragyan).
under the Project Sampark.

www.shankariasacademy.com | www.iasparliament.com
55

 The Rover Pragyan is housed inside Vikram  Similarly, a former High Court Judge can also
lander. become a State Human Rights Commission
Chairperson besides a High Court Chief
 The mission objective of Chandrayaan-2 is to Justice.
develop and demonstrate the key technologies
for end-to-end lunar mission capability, 9. c
including soft-landing and roving on the lunar
10. c
surface.
 Ministry of Tourism under the Swadesh
 On the science front, this mission aims to
Darshan Scheme has identified Tirthankar
further expand our knowledge about the Moon
Circuit as one of the fifteen thematic circuits
through a detailed study of its topography,
for development in the country.
mineralogy, surface chemical composition,
thermo-physical characteristics and  All the sites associated with Jainism are
atmosphere leading to a better understanding covered under this circuit.
of the origin and evolution of the Moon.
7. d
24-07-2019
 The Ministry of Culture has embarked
upon an ambitious project of the digitization 1) Consider the following statements with
of the collections of the Museums. respect to the UKEIRI Mobility Programme:
Study in India
 In this endeavour, the Ministry of Culture,
through the technical expertise of Centre for 1. It aims to generate up to 200
Development of Advanced Computing (C – opportunities for undergraduate
DAC) Pune and the Art Institute of Chicago got students at UK universities to visit India
standardized a software entitled "JATAN" for by March 2021.
implementation in its Museums. 2. The programme will be funded by the
UK and Indian governments as part of
 The twin purpose of the project is making
the UK-India Education Research
effective utilization of technology in museum
Initiative (UKEIRI).
management and bringing the collections of
these museums closer to the public by making Which of the statement(s) given above is/are
them available for online viewing over the correct?
internet.
a. 1 only
8. c
b. 2 only
 The Parliament has recently passed the
c. Both 1 and 2
Protection of Human Rights (Amendment)
Bill, 2019. d. Neither 1 nor 2
 The Bill amends the Protection of Human
Rights Act, 1993.
2) Consider the following statements with
 It provides for the reduction of the term of respect to National Health Accounts Estimates
the Chairperson and Members of the 2015-16
Commission and the State Commissions 1. Jammu and Kashmir and Assam spent
from five to three years and shall the largest fraction of their Gross State
be eligible for re-appointment. Domestic Product (GSDP) on public
 The bill also states that besides the former health.
Chief Justice of India, as is the current 2. Uttar Pradesh and Bihar have spent
requirement, a former Supreme Court least of their GSDPs on public health.
Judge can also become the NHRC
Chairperson. Which of the statement(s) given above is/are
correct?

www.shankariasacademy.com | www.iasparliament.com
56

a. 1 only 6) Consider the following statements with


respect to Global Innovation Index (GII) 2019
b. 2 only
1. It has been jointly developed by the
c. Both 1 and 2
World Ecomic Forum (WEF), Cornell
d. Neither 1 nor 2 University and Paris-based business
school Insead.
2. The theme for the year 2019 is Creating
3) Tiangong-2 often seen in the news recently Healthy Lives - The Future of Medical
is a Innovation.
a. Space Station 3. India has been ranked the most
b. Space Telescope innovative country in the Central and
Southern Asia Region every year since
c. Super Computer 2011.
d. None of the above Which of the statement(s) given above is/are
correct?
4) Consider the following statements a. 3 only
1. He was the chief strategist of the b. 1 and 2 only
Hindustan Socialist Republican c. 2 and 3 only
Association (HSRA).
d. 1, 2 and 3
2. He became very popular after the
Kakori Train robbery in 1925 and the
killing of the assistant superintendent 7) Consider the following statements
Saunders in 1928.
1. A person who is appointed to be a
3. Bhagat Singh joined with him following governor of a state should be an
the death of Lala Lajpat Rai. outsider, that is, he/she should not
Identify the correct personality using belong to the respected state.
the description given above: 2. While appointing the governor, the
a. Sachin Sanyal president is required to consult the
chief minister of the state concerned.
b. Ramprasad Bismil
Which of the above is/are mentioned in Indian
c. Jogesh Chandra Chatterjee Constitution as the qualification for the
d. Chandra Shekhar Azad appointment of a person as a Governor of a
State?
a. 1 only
5) The oceanic pole of inaccessibility often
called as The Point Nemo refers to? b. 2 only
a. The most remote location on Earth c. Both 1 and 2
b. The most remote location on Atlantic Ocean d. Neither 1 nor 2
c. An imaginary line joining the inaccessible
areas of the planet Earth 8) Consider the following statements
d. The most remote location on Moon 1. He was an Indian social reformer and
freedom activist.
2. He was also called the „Father of Indian
Unrest‟.

www.shankariasacademy.com | www.iasparliament.com
57

3. He was the author of the book “Gita  It aims to generate up to 200 opportunities for
Rahasya”. undergraduate students at UK universities
to visit India by March 2021.
Identify the correct personality using
the description given above:  Priority for the programme to visit India
a. Rash Behari Ghosh during their courses will be given to students
from traditionally underrepresented groups.
b. Bal Gangadhar Tilak
 The programme will be funded by the UK
c. Madan Mohan Malaviya and Indian governments as part of Phase
d. None of the above 3 of the UK-India Education Research
Initiative (UKEIRI) and delivered by the
British Council, Universities UK International
9) Consider the following statements with and EdCIL in India.
respect to the Tenth Schedule of the Indian 2. a
Constitution
 Jammu and Kashmir and Assam spent
1. If the member voluntarily gives up the the largest fraction of their Gross State
membership of the party, he shall be Domestic Product (GSDP) on public health,
disqualified. according to National Health Accounts
2. If there is a merger between two Estimates 2015-16, the Ministry of Health
political parties and one-third of the and Family Welfare has told the Parliament
members of a legislature party agree to recently.
the merger, they will not be disqualified.  The share of Government Health Expenditure
Which of the statement(s) given above is/are (GHE) in the GSDPs of J&K and Assam were
correct? 1.7% and 1.5% respectively, the data show.

a. 1 only  Haryana and Maharashtra are towards


the lower end of the table, having spent 0.6%
b. 2 only and 0.7% of their GSDPs on health
c. Both 1 and 2 respectively.

d. Neither 1 nor 2 3. a
 Tiangong-2 was a manned Chinese space
station that was destroyed upon its controlled
10) Markanda and Tangri are the tributaries of re-entry into the Earth‘satmosphere over the
which of the following rivers? Pacific Ocean on July 19.
a. Son  Tiangong-2 was retired from service after it
b. Gomti had completed its experiments in space.
c. Mahanadi  Tiangong-2 was 10.4 metres long and 3.35
metres wide at its widest point, and weighed
d. Ghaggar 8.6 metric tonnes.
 It was launched on September 15, 2016 and, in
Answers late 2016, hosted two Chinese astronauts for
30 days in what was China‘s longest manned
1. c space mission so far.
 The ―UKEIRI Mobility Programme: Study in 4. d
India‖, an initiative of Universities UK
International (UUKI) and British  The revolutionary activity in Punjab, United
Council India, was launched recently. Provinces and Bihar was dominated by
the Hindustan Republican
Association/Army or HRA.

www.shankariasacademy.com | www.iasparliament.com
58

 The HRA was founded in October 1924 in  India has been ranked the most innovative
Kanpur by Ramprasad Bismil, Jogesh country in the Central and Southern Asia
Chandra Chatterjee and Sachin Sanyal. Region every year since 2011.
 Under the leadership of Chandra Shekhar  India has consistently outperformed on
Azad, the name of HRA was changed to innovation relative to its Gross Domestic
Hindustan Socialist Republican Product (GDP) per capita.
Association (HSRA) after the death of its
7. d
founder, Ram Prasad Bismil, and three other
prominent party leaders.  The Constitution lays down only two
qualifications for the appointment of a
 The participants included Bhagat Singh,
person as a governor. These are:
Sukhdev, Bhagwaticharan Vohra from Punjab.
1. He should be a citizen of India.
 Bhagat Singh joined Azad following the death
of Lala Lajpat Rai, after he was beaten by 2. He should have completed the age of 35 years.
police officials.
 Additionally, two conventions have also
 Azad trained Singh and others in covert developed in this regard over the years.
activities.
1. He should be an outsider, that is, he should not
5. a belong to the state where he is appointed, so
that he is free from the local politics.
 Point Nemo, officially known as ―the oceanic
pole of inaccessibility,‖ or, more simply 2. While appointing the governor, the president
put, the point in the ocean that is farthest away is required to consult the chief minister of the
from land. state concerned, so that the smooth
functioning of the constitutional machinery in
 The spot is quite literally the middle of the state is ensured.
nowhere, surrounded by more than 1,000
miles of ocean in every direction.  However, both the conventions have been
violated in some of the cases.
 Point Nemo is located over 1,000 miles
(1,600km) equidistantly from the coasts of 8. b
three far-flung islands.  Bal Gangadhar Tilak, commonly known as
 Ducie Island (one of the Pitcairn islands) is to Lokamanya Tilak was a leader of the Indian
the north, Motu Nui (of the Easter Island independence movement and belonged to the
chain) is to the north-east and Maher Island extremist faction.
(off the coast of Antarctica) is to the south.  He was also called the ‗Father of Indian
 Most of Tiangong-2 burnt up in the Unrest‘.
atmosphere, and the remaining debris fell
 Gita Rahasya and the Arctic Home of the
near Point Nemo, the most remote location on Vedas were the books written by Tilak.
Earth, which is so far from land.
9. a
6. c
 There are two grounds on which a member of a
 The GII has been jointly developed by legislature can be disqualified.
the WIPO, Cornell University and Paris-based
business school Insead.  One, if the member voluntarily gives up
the membership of the party, he shall
 The theme of the 2019 GII is Creating Healthy be disqualified.
Lives - The Future of Medical Innovation,
which aims to explore the role of medical  Voluntarily giving up the membership is not
innovation as it shapes the future of the same as resigning from a party.
healthcare.
 Even without resigning, a legislator
can be disqualified if by his conduct the
Speaker/Chairman of the concerned House

www.shankariasacademy.com | www.iasparliament.com
59

draws a reasonable inference that the member 2) Consider the following statements with
has voluntarily given up the membership of his respect to National Institute of Miners' Health
party. (NIMH)
 Second, if a legislator votes in the House 1. It conducts applied research in
against the direction of his party and his action occupational health and hygiene and
is not condoned by his party, he can be specializes in providing technical
disqualified. support services to mining and mineral
based Industry.
 However, there is an exception that was
provided in the law to protect the legislators 2. It is an autonomous institute works
from disqualification. under the Indian Council of Medical
Research (ICMR).
 The 10th Schedule says that if there is a merger
between two political parties and two- Which of the statement(s) given above is/are
thirds of the members of a legislature party correct?
agree to the merger, they will not be a. 1 only
disqualified.
b. 2 only
10. d
c. Both 1 and 2
 Ghaggar originates from Dagshai village in
Himachal‘s Solan district and flows through d. Neither 1 nor 2
Haryana, Punjab, and then ends in Rajasthan.
 The river and its tributaries — Markanda 3) With respect to Fair and Remunerative
and Tangri — pass from Kalka, Panchkula, Price (FRP) of sugarcane, consider the
Ambala, Kaithal, (in Haryana) and then enters following statements:
Punjab‘s Patiala, Sangrur districts.
1. The FRP of sugarcane is determined
under Sugarcane (Control) Order, 1966.
25-07-2019 2. The FRP is determined based on the
recommendation of the Commission of
1) Consider the following statements with
Agricultural Costs & Prices (CACP).
respect to National Institute of Occupational
Health (NIOH) Which of the statement(s) given above is/are
correct?
1. It aims to promote intensive research to
evaluate environmental stresses/factors a. 1 only
at the workplace.
b. 2 only
2. It is an autonomous institute works
c. Both 1 and 2
under the Ministry of Labour.
d. Neither 1 nor 2
3. It was dissolved recently and merged
with the National Institute of Miner‟s
Health (NIMH).
4) Consider the following statements with
Which of the statement(s) given above is/are respect to North Eastern States Road
correct? Investment Programme (NESRIP) project
a. 1 only 1. It envisages the construction/up-
gradation of roads in six North-Eastern
b. 1 and 2 only
States.
c. 2 and 3 only
2. The project was assisted by the World
d. 1, 2 and 3 Bank.
3. National Highways Authority of India
(NHAI) is the executing agency

www.shankariasacademy.com | www.iasparliament.com
60

responsible for overall coordination and a. Russia and South Korea


monitoring of progress.
b. China and Philippines
Which of the statement(s) given above is/are
c. Russia and Japan
correct?
d. South Korea and Japan
a. 1 only
b. 2 and 3 only
9) Arrange the following from South to North
c. 1, 2 and 3
1. East Sea
d. None
2. East China Sea
3. South China Sea
5) Consider the following pairs
4. Sea of Okhotsk
1. Rokhia Power Project – Tripura
Select the correct answer using the codes given
2. Gumti Hydro Electric Project – Uttar
below:
Pradesh
a. 4-1-2-3
Which of the pair(s) given above is/are
correctly matched? b. 3-2-1-4
a. 1 only c. 3-1-2-4
b. 2 only d. 4-3-2-1
c. Both 1 and 2
d. Neither 1 nor 2 10) Consider the following statements
regarding the concept of Eclipse
1. When the shadow of the Earth falls on
6) Union Finance Minister has recently
the moon it is called Lunar eclipse.
launched the ―Taxalogue‖. It refers to?
2. When the shadow of the moon falls on
a. e-journal
the Earth it is called Solar eclipse.
b. Tax calculation app
Which of the statements given above is/are
c. Publicity Kit for tax awareness correct?
d. Awareness film about CBDT a. 1 only
b. 2 only
7) Motihari-Amlekhgunj oil c. Both 1 and 2
pipeline sometimes seen in the news recently is
d. Neither 1 nor 2
a transnational pipeline between which of the
following two countries?
a. India and Bhutan Answers
b. India and Nepal 1. a
c. India and Bangladesh  The National Institute of Occupational
d. India and Afghanistan Health (NIOH) is the premier institute,
under the aegis of the Indian Council of
Medical Research (ICMR) under the
8) Dokdo/Takeshima islands sometimes seen Department of Health Research, Ministry of
in the news recently is a disputed territory Health and Family Welfare.
between which of the following two countries?

www.shankariasacademy.com | www.iasparliament.com
61

 The Union Cabinet has recently approved  It is assisted by Asian Development


to dissolve National Institute of Miners' Bank (ADB).
Health (NIMH), an autonomous Institute
under Ministry of Mines (MoM) and merge /  Ministry of Development of North
amalgamate with ICMR-National Eastern Region (MDoNER) is the
Institute of Occupational Health Executing Agency responsible for overall
(NIOH), Ahmedabad, Ministry of Health & coordination with ADB and State Governments
Family Welfare (MoH&FW) with all assets and monitoring of progress.
and liabilities. 5. a
2. a  Rokhia Power Project is located at
 National Institute of Miners‘ Health West Tripura district whereas Gumti Hydro
(NIMH) is an autonomous Institute Electric Project is in Gomati district
under Ministry of Mines. of Tripura.

 It conducts applied research in occupational  Asian Development Bank has recently


health and hygiene and specializes in sanctioned 1,925 crore rupees project for up-
providing technical support services to mining gradation of power generation and
and mineral based Industry. distribution in Tripura.
6. a
 The Union Cabinet has recently approved
to dissolve National Institute of Miners'  Union Finance Minister has recently launched
Health (NIMH), an autonomous Institute an in-house e-journal ―Taxalogue‖
under Ministry of Mines (MoM) and merge / or Dialogue on Taxes, which is a new
amalgamate with ICMR-National initiative taken by CBDT being a quarterly
Institute of Occupational Health e-journal where one can contribute articles
(NIOH), Ahmedabad, Ministry of Health & on issues pertaining to taxation.
Family Welfare (MoH&FW) with all assets
and liabilities. 7. b

3. c  The 69 km long Motihari-Amlekhgunj oil


pipeline has been constructed by India in 15
 The Fair and Remunerative Price (FRP) months, half of its 30 months deadline.
of sugarcane is determined under Sugarcane
(Control) Order, 1966.  It will ensure smooth, cost-effective and
environment-friendly supply of petroleum
 This will be uniformly applicable all over products to Nepal.
the country.
 It is the first transnational petroleum pipeline
 The FRP is based on the recommendation of from India and first South Asian oil pipeline
the Commission of Agricultural Costs & corridor.
Prices (CACP) as per its report of August
2018 on the price policy for sugarcane for the  It is also the first oil pipeline in Nepal.
2019-20 season.  The prestigious project is expected to be
 The CACP has recommended the same price inaugurated soon by the Prime Minister
for the 2019-20 sugar season as it was for the Narendra Modi and his Nepalese counterpart
sugar season 2018-19. K P Sharma Oli.

4. a 8. d

 North Eastern States Roads Investment  South Korean media has recently reported that
Programme (NESRIP) envisaged a Russian warplane recently violates its
construction/up-gradation of total 433.425 km airspace.
long roads in 6 North Eastern States of  The alleged incursion happened over the
Assam, Manipur, Meghalaya, Mizoram, Sikkim disputed Dokdo/Takeshima islands,
and Tripura.

www.shankariasacademy.com | www.iasparliament.com
62

which are occupied by South Korea but


also claimed by Japan.
2) Consider the following statements with
9. b respect to Repco Bank
 North Korea has recently launched two short- 1. It is a multi state cooperative society
range missiles. The missiles were fired from established in 1969 for the
Wonsan in eastern North Korea and landed in rehabilitation of repatriates from
the Sea of Japan. Burma and Sri Lanka.
 Russia‟s plane was taking part in a joint air 2. It comes under the administrative
patrol by Russian and Chinese warplanes control of Ministry of Home Affairs.
over the Sea of Japan and the East China Which of the statement(s) given above is/are
Sea - the first-ever air patrol between the two correct?
countries.
a. 1 only
 South Korean media has recently reported that
a Russian warplane recently violates its b. 2 only
airspace. c. Both 1 and 2
10. c d. Neither 1 nor 2
 An eclipse is an astronomical event that occurs
when an astronomical object is temporarily
obscured, either by passing into the shadow of 3) Consider the following statements
another body or by having another body pass 1. Wasted children are those who have low
between it and the viewer. height for their age
 The term eclipse is most often used to describe 2. It is a reflection chronic under nutrition
either a solar eclipse, when the Moon's shadow
crosses the Earth's surface, or a lunar eclipse, Which of the following statement(s) is/are
when the Moon moves into the Earth's correct?
shadow. a. 1 only
b. 2 only
26-07-2019 c. Both 1 and 2
1) Consider the following statements with d. Neither 1 nor 2
respect to UN-SPIDER
1. It was established in 2006 under the
United Nations Office for Outer Space 4) Consider the following statements with
Affairs (UNOOSA). respect to Van Dhan Vikas Karyakram

2. It develops solutions to address the 1. It aims to tap into the traditional


limited access developing countries knowledge of tribal people and to
have to specialized technologies that can convert the tribal wisdom into a
be essential in the management of remunerative economic activity.
disasters and the reducing of disaster 2. Implementation of Van Dhan Vikas
risks. Karyakram is through Tribal
Which of the statement(s) given above is/are Cooperative Marketing Development
correct? Federation of India (TRIFED).

a. 1 only Which of the statement(s) given above is/are


correct?
b. 2 only
a. 1 only
c. Both 1 and 2
b. 2 only
d. Neither 1 nor 2

www.shankariasacademy.com | www.iasparliament.com
63

c. Both 1 and 2 2. Only male rhinoceros have horns.


d. Neither 1 nor 2 Which of the above statement(s) is/are
correct?
a. 1 only
5) Consider the following statements with
respect to the International Charter on Space b. 2 only
and Major Disasters
c. Both 1 and 2
1. It is a worldwide collaboration among
d. Neither 1 nor 2
space agencies, through which satellite-
derived information and products are
made available to support disaster
response efforts. 8) Consider the following statements about
Parliamentary committees
2. Only agencies that possess and are able
to provide satellite-based Earth 1. They draw their authority from Article
Observation data can be members of the 105 and Article 118.
International Charter. 2. Parliament is not bound by the
Which of the statement(s) given above is/are recommendations of committees.
correct? Which of the above statement(s) is/are
a. 1 only correct?

b. 2 only a. 1 only

c. Both 1 and 2 b. 2 only

d. Neither 1 nor 2 c. Both 1 and 2


d. Neither 1 nor 2

6) Consider the following statements with


respect to RTI (Amendment) Bill, 2019 which 9) Namdapha Tiger Reserve is located in which
was passed recently by the Parliament of the following states?
1. It seeks to amend the Right to a. Assam
Information Act, 2005.
b. Meghalaya
2. It seeks to empower the Centre to make
rules to decide the tenure, salary of c. Jammu and Kashmir
Information Commissioners of the d. Arunachal Pradesh
Chief Information Commissioners and
also of State Information
Commissioners. 10) Consider the following statements with
Which of the statement(s) given above is/are respect to
correct? 1. Tarapur Atomic Power Plant –
a. 1 only Maharashtra

b. 2 only 2. Kudankulam Atomic Power Plant –


Tamil Nadu
c. Both 1 and 2
3. Kaiga Atomic Power Plant – Karnataka
d. Neither 1 nor 2
Which of the statement(s) given above is/are
correct?
7) Consider the following statements about one a. 2 only
horned rhinoceros
b. 1 and 2 only
1. They are found only in India and Nepal.

www.shankariasacademy.com | www.iasparliament.com
64

c. 2 and 3 only among space agencies, through which satellite-


derived information and products are made
d. 1, 2 and 3
available to support disaster response efforts.
 Only agencies that possess and are able to
Answers provide satellite-based Earth Observation data
can be members of the International Charter.
1. c
 The members cooperate on a voluntary
 The United Nations Platform for Space-based basis.
Information for Disaster Management and
Emergency Response (UN-SPIDER) was 6. c
established in 2006 under the United Nations 7. a
Office for Outer Space Affairs (UNOOSA).
 Both male and female rhinoceros have horns,
 UN-SPIDER develops solutions to address the but not newborn young.
limited access developing countries have to
specialized technologies that can be essential 8. c
in the management of disasters and the
 Parliamentary committees draw their
reducing of disaster risks.
authority from Article 105 (on privileges of
2. c Parliament members) and Article 118 (on
Parliament‘s authority to make rules for
 Repco Bank is a multi state cooperative society regulating its procedure and conduct of
established in 1969 by the Government of business).
India for the rehabilitation of repatriates from
Burma and Sri Lanka and is under the 9. d
administrative control of Ministry of Home 10. d
Affairs.
1. Tarapur Atomic Power Plant – Maharashtra
3. d
2. Kudankulam Atomic Power Plant – Tamil
 Wasted children are those who have low Nadu
weight for their height, reflecting acute under
nutrition 3. Kaiga Atomic Power Plant – Karnataka

 Stunted children are those who have low


height for their age, reflecting chronic under
27-07-2019
nutrition
1) Consider the following statements with
4. a
respect to Agricultural Marketing
 Van Dhan Vikas Karyakram aims to tap Infrastructure (AMI) Scheme
into the traditional knowledge and skill sets of 1. It is a demand driven interest subsidy
tribal people by adding technology and scheme.
Information Technology for up-gradation of
output at each stage and to convert the tribal 2. It is a sub-scheme of Integrated Scheme
wisdom into a remunerative economic activity. for Agricultural Marketing (ISAM).
 Van Dhan Vikas Karyakram seeks to promote 3. Assistance under the AMI scheme is
and leverage the collective strength of tribal available only to individual farmers.
people to achieve a viable scale. Which of the statement(s) given above is/are
 Implementation of Van Dhan Vikas Karyakram correct?
is through Van Dhan Kendras. a. 2 only
5. c b. 1 and 2 only
 The International Charter on "Space and c. 2 and 3 only
Major Disasters" is a worldwide collaboration
d. 1, 2 and 3

www.shankariasacademy.com | www.iasparliament.com
65

2. Under the agreement, Iraq agreed to


restrict its capacity to enrich uranium.
2) Consider the following statements with
respect to Green Revolution– Krishonnati 3. Missile launches are not forbidden
Yojana under the deal.
1. It is an umbrella scheme consist of 12 Which of the statement(s) given above is/are
schemes comprising both Central Sector correct?
as well as Centrally Sponsored Schemes.
a. 1 only
2. National Food Security Mission (NFSM)
b. 1 and 2 only
is one of the mission under the
Krishonnati Yojana. c. 1 and 3 only
Which of the statement(s) given above is/are d. 1, 2 and 3
correct?
a. 1 only
6) Consider the following statements
b. 2 only
1. Elephant Census in India will be
c. Both 1 and 2 conducted once in 5 years under the
aegis of Project Elephant.
d. Neither 1 nor 2
2. According to 2017 Census, Kerala has
the highest number of elephants and
3) A high-level 10-member Committee has been Maharashtra has the least number of
constituted recently under the Chairmanship elephants.
of Kiren Rijiju for which of the following
Which of the statement(s) given above is/are
purposes?
correct?
a. To coordinate and strategise the preparation
a. 1 only
for the 2020 and 2024 Olympics
b. 2 only
b. To draft a plan to ease the entry of rural
talented youth in to the sports arena c. Both 1 and 2
c. To examine the issues and to reconstitute the d. Neither 1 nor 2
Sports Authority of India
d. None of the above
7) Why a knotweed Polygonum molle native to
Himalayas is often mentioned in the news?
4) Ream Naval Base sometimes seen in the a. It absorbs the pollutants in the atmosphere
news recently is located in? and helps to clean the air in the environment
a. Hudson Bay b. Its extract is widely used in the treatment of
Barotrauma which is caused by a decrease in
b. Gulf of Aden
air pressure
c. Gulf of Thailand
c. It is invasive and reduces the availability of
d. Gulf of Carpentaria land for the native species
d. None of the above
5) Consider the following statements with
respect to Joint Comprehensive Plan of Action
8) LightSail 2 sometimes seen in the news
(JCPOA)
recently is a
1. It was signed between Iran and the P5+1
a. Space craft powered purely by the sunlight
Countries.

www.shankariasacademy.com | www.iasparliament.com
66

b. Intercontinental ballistic missile launched by 2. c


Iran
‗Green Revolution– Krishonnati Yojana‘ is an Umbrella
c. NASA‘s satellite to study the asteroid Hektor Scheme comprising both Central Sector as well as
Centrally Sponsored Schemes/Missions.
d. Stratospheric Observatory for Infrared
Astronomy This Umbrella Scheme has the following twelve
Schemes/Missions:
1. Mission for Integrated Development of
9) Naegleria Fowleri sometimes seen in the
Horticulture (MIDH);
news recently is a
2. National Mission on Oil Seeds and Oil Palm
a. Bonding Hormone
(NMOOP);
b. Brain eating amoeba
3. National Food Security Mission (NFSM)
c. Carcinogenic agent
4. National Mission for Sustainable Agriculture
d. Snake species endemic to Lakshadweep (NMSA);
5. Sub-Mission on Agriculture Extension
(SMAE);
10) Consider the following pairs
6. Sub-Mission on Seeds & Planting Material
1. Dighi Port – Madhya Pradesh (SMSP);
2. Mormugao Port – Goa 7. Sub-Mission on Agricultural Mechanisation
3. Paradip Port – West Bengal (SMAM);
Which of the pair(s) given above is/are 8. Sub-Mission on Plant Protection and Plant
correctly matched? Quarantine (SMPPQ);
a. 2 only 9. Integrated Scheme on Agricultural Census,
Economics and Statistics;
b. 1 and 3 only
10. Integrated Scheme on Agricultural
c. 2 and 3 only Cooperation;
d. 1, 2 and 3 11. Integrated Scheme on Agricultural Marketing
(ISAM);

Answers 12. National e-Governance Plan in Agriculture


(NeGP-A).
1. a
3. a
 Agricultural Marketing Infrastructure (AMI) is
a demand driven scheme.  A high-level 10-member Committee has been
constituted recently under the Chairmanship
 AMI Scheme is a sub-scheme of Integrated of Sports Minister Kiren Rijiju to
Scheme for Agricultural Marketing coordinate and strategise the
(ISAM). preparation for the 2020 and 2024
Olympics.
 AMI scheme is a back ended capital subsidy
scheme in which rate of subsidy is 25% and  The objective of the committee is to ensure
33.33% based on the category of eligible that the performance of the Indian athletes is
beneficiary and is provided on capital cost of optimised in the Olympics and other
the project. multidisciplinary events.
 Assistance under the AMI sub-scheme is 4. c
available to individual farmers, groups of
farmers/growers, registered Farmer  A recent report has alleged that China has
signed a secret deal to allow its military troops
Producer Organizations (FPOs) etc.

www.shankariasacademy.com | www.iasparliament.com
67

to be stationed at Ream Naval Base on the  LightSail 2 is the first steerable solar
Gulf of Thailand. sail ever launched into Earth‘s orbit.
5. c  Most rockets burn their onboard fuel to
accelerate in space and then coast on their
 Joint Comprehensive Plan of Action
built-up momentum.
(JCPOA) is an agreement reached
between Iran and P5+1 countries (The US,  Solar sails, on the other hand, are powered by
France, The UK, China, Russia and Germany). photons emitted by the sun.
 As part of the deal, Iran (Not Iraq) agreed to  They accelerate very slowly, but since their
reduce its number of centrifuges - tube-shaped thrust is continuous, they can conceivably
machines that help enrich uranium- by two achieve tremendous speeds.
thirds.
9. b
 It also agreed to reduce its stockpile of
enriched uranium by 98% and limit uranium  Naegleria Fowleri is a single celled
enrichment to 3.67%. organism known as the brain eating
amoeba.
 Missile launches are not forbidden under
the 2015 nuclear deal.  It is a heat-loving amoeba, known for
striking swimmers during sweltering summer
 But, a UNSC Resolution, passed just as the temperatures (it prefers temps up to
agreement was reached, says that ―Iran is 115°F/46°C).
called upon not to undertake any activity
related to ballistic missiles designed to be 10. a
capable of delivering nuclear weapons, 1. Dighi Port – Maharashtra
including launches using such ballistic missile
technology. 2. Mormugao Port – Goa

 Iran fired a Shahab – 3 medium range missile 3. Paradip Port – Odisha


recently.
6. a 29-07-2019
 Elephant census will be conducted once in 5 1) Consider the following statements with
years under the aegis of Project Elephant. respect to the term Green Bonus sometimes
 The last elephant census was conducted seen in the news recently
in 2017. 1. It denotes the money which is to be
7. c given for the effort made by a country,
state or community for preservation of
 The knotweed, known as Polygonum green cover.
molle (Sikkim knotweed), which has been
2. The contribution to the fund was made
recently reclassified as Koenigia mollis, has
jointly by the Green Climate Fund (GCF)
begun spreading along many streams and
and UN Environment Programme
rivulets in the upper Nilgiris, especially
(UNEP).
around Doddabetta, Kodappamund, Adasolai
and the Ketti Valley. Which of the statement(s) given above is/are
correct?
 It is native to the Himalayas and its
spread has potentially threatening a. 1 only
biodiversity along streams and rivers.
b. 2 only
 It can cover huge expanses of land, leaving
c. Both 1 and 2
little to no space for other native plant species
to grow. d. Neither 1 nor 2
8. a

www.shankariasacademy.com | www.iasparliament.com
68

2) Consider the following statements with c. Both 1 and 2


respect to Iconic Tourist Sites Initiative
d. Neither 1 nor 2
1. It aims to develop 17 Iconic Tourist Sites
in the country in to world class tourist
destinations, to serve as a model for 5) Consider the following statements with
other tourism sites. respect to 2018 Tiger Census (All India Tiger
Estimation 2018)
2. Ministry of Tourism will be the nodal
agency for this project. 1. At present India has the distinction of
having the maximum number of tigers
Which of the statement(s) given above is/are
in the world.
correct?
2. According to the census, India has
a. 1 only
achieved the target of doubling the tiger
b. 2 only population four years early ahead of
2022.
c. Both 1 and 2
Which of the statement(s) given above is/are
d. Neither 1 nor 2
correct?
a. 1 only
3) Consider the following statements with
b. 2 only
respect to Hepatitis
c. Both 1 and 2
1. It is an inflammation of the liver and
Hepatitis viruses are the only cause of d. Neither 1 nor 2
hepatitis in the world.
2. There are 5 main hepatitis viruses,
6) Consider the following statements with
referred to as types A, B, C, D and E.
respect to Banning of Unregulated Deposit
3. There is no vaccine available against the Schemes Bill 2019
Hepatitis B.
1. It seeks to provide for a comprehensive
Which of the statement(s) given above is/are mechanism to ban the unregulated
correct? deposit schemes other than deposits
taken in the ordinary course of
a. 2 only
businesses.
b. 1 and 2 only
2. The bill provides for attachment of
c. 1, 2 and 3 properties or assets and subsequent
realization of assets for repayment to
d. None depositors.
Which of the statement(s) given above
4) Consider the following statements with is/are incorrect?
respect to Mozambique a. 1 only
1. It is a landlocked country located in b. 2 only
Southern Africa.
c. Both 1 and 2
2. Tropic of Capricorn passes through this
country. d. Neither 1 nor 2
Which of the statement(s) given above
is/are incorrect?
7) Benin is a country which shares its border
a. 1 only with which of the following?
b. 2 only a. Gulf of Guinea

www.shankariasacademy.com | www.iasparliament.com
69

b. Great Australian Bight d. Uttar Pradesh


c. Antongil Bay
d. Bay of Biscay Answers
1. a
8) With respect to 2011 Census, consider the  Green Bonus is a new term in the
following statements environmental parlance denoting the money
1. Migration to cities is mostly intra-state which is to be given for the effort made by a
rather than inter-state. country, state or community for preservation
of green cover.
2. Inter-State migration to the big cities
often tends to be intra-regional, mostly  It is a compensation to be given to the people
driven by migrants‟ desire to be close to for the sacrifices they have made in preserving
home. the green cover, which not only benefits them,
but also benefits others nearby and humanity
Which of the statement(s) given above is/are as a whole.
correct?
 Eleven States belonging to the
a. 1 only Himalayan Region recently sought a
b. 2 only „Green Bonus‟ from the Centre considering
their contributions to environmental
c. Both 1 and 2 conservation.
d. Neither 1 nor 2  The Himalayan states were also at a
disadvantage because large swathes of land fell
into eco-sensitive zones where all sorts of
9) Consider the following statements with development activities could not be carried
respect to PM Vaya Vandana Yojana (PMVVY) out.
1. It is a pension scheme offered through 2. c
the State Bank of India (SBI).
 Union Finance Minister in her recent budget
2. There is no upper limit on the age for speech has said that 17 ―Iconic Tourist Sites‖ in
subscription. the country would be developed by the
3. Only those who have completed 60 government ―into world class tourist
years of age are eligible to subscribe to destinations, to serve as a model for other
it. tourism sites‖.
Which of the statement(s) given above is/are  The initiative was aimed at enhancing India‘s
correct? soft power.
a. 3 only  The Tourism Ministry will be the nodal
agency for this initiative.
b. 1 and 2 only
3. a
c. 2 and 3 only
d. 1, 2 and 3  Hepatitis is an inflammation of the liver.
 The condition can be self-limiting or can
progress to fibrosis (scarring), cirrhosis or liver
10) Pilibhit Tiger Reserve is located in which of cancer.
the following states?
 Hepatitis viruses are the most common cause
a. Mizoram of hepatitis in the world but other infections,
b. Karnataka toxic substances (e.g. alcohol, certain drugs),
and autoimmune diseases can also cause
c. Maharashtra hepatitis.

www.shankariasacademy.com | www.iasparliament.com
70

 There are 5 main hepatitis viruses, referred to  The seven-day visit is the first high-level tour
as types A, B, C, D and E. of any Head of State from India to the three
West African countries- Benin, The Gambia
 Hepatitis C is more dangerous than Hepatitis and Guinea.
B, because there is no vaccination against it
– yet.  Its southern coastline is a part of the Gulf of
Guinea in the northernmost tropical portion of
4. a
the Atlantic Ocean.
5. c
8. c
 Indian PM has recently released the ―All
 The 2011 Census data shows that much of the
India Tiger Estimation 2018‖ on the
migration to the big cities is intra-State
occasion of Global Tiger Day in New Delhi.
rather than inter-State.
 According to the survey, the count of tigers in
 It also show that inter-State migration to the
India has risen to 2,967.
big cities often tends to be intra-regional,
 India has achieved the target of doubling mostly driven by migrants‘ desire to be close to
the tiger population four years early ahead of home.
2022.
 For instance, Chennai, Bengaluru and
 India is now home to nearly 3,000 tigers, a Hyderabad have a greater proportion of
third more than it had four years ago. migrants from the South than from the North.

 India is now one of the biggest and most 9. c


secure habitats of tiger and is estimated to be  PM Vaya Vandana Yojana (PMVVY) is a
home to around 70 percent of the world's pension scheme offered through the Life
tigers. Insurance Corporation of India (LIC).
 Due to concerted efforts under Project Tiger, at  Only those who have completed 60
present India has the distinction of having years of age are eligible to subscribe to it.
the maximum number of tigers in the
world.  There is no upper limit on the age for
subscription.
6. d
 It provides regular payout to those who invest
 Parliament has recently passed the Banning
in the scheme.
of Unregulated Deposit Schemes Bill
2019. 10. d
 The bill seeks to provide for a comprehensive  Pilibhit Tiger Reserve is one among the
mechanism to ban the unregulated deposit two tiger reserves in the country -- the other is
schemes other than deposits taken in the the Tadoba-Andhari reserve in Maharashtra --
ordinary course of businesses and to protect that has seen increased man-animal
the interest of depositors. conflict in recent years.
 The bill provides for attachment of properties
or assets and subsequent realisation of assets
for repayment to depositors. 30-07-2019
 The bill also proposes adequate provisions for 1) Consider the following statements with
repayment of deposits in cases where such respect to LCU L-56 sometimes seen in the
schemes nonetheless manage to raise deposits news recently
illegally. 1. It is a Light Combat Utility Helicopter
7. a inducted recently in to the Indian Air
Force (IAF).
 President Ram Nath Kovind has
2. Its primary role is to transport and
reached Benin on the first leg of his three
deploy main battle tanks, armoured
nations' tour to the West African region.

www.shankariasacademy.com | www.iasparliament.com
71

vehicles, troops and equipment from


ship to shore.
4) Consider the following statements with
Which of the statement(s) given above is/are respect to Kodaikanal Malai
correct? Poondu (Kodaikanal Hill Garlic)
a. 1 only 1. It is grown in the Kodaikanal Hills of
Tamilnadu.
b. 2 only
2. It is known for its medicinal and
c. Both 1 and 2
preservative properties.
d. Neither 1 nor 2
3. The Geographical Indications Registry
has recently granted the Geographical
Indication (GI) tag to Kodaikanal Malai
2) Consider the following statements with Poondu.
respect to Grain by Grain Report sometimes
seen in the news recently Which of the statement(s) given above is/are
correct?
1. It is the complete assessment of the
environmental performance of fertilizer a. 1 only
industry in India.
b. 1 and 2 only
2. It was released recently by the NITI
c. 2 and 3 only
Aayog.
d. 1, 2 and 3
Which of the statement(s) given above is/are
correct?
a. 1 only 5) Consider the following statements with
respect to All India Tiger Estimation - 2018
b. 2 only
1. In India, Tiger Census will be conducted
c. Both 1 and 2
once in every 5 years.
d. Neither 1 nor 2
2. Karnataka is having the highest number
of tigers in India followed by Madhya
Pradesh.
3) Consider the following statements with
respect to National Commission of Minorities Which of the statement(s) given above is/are
(NCM) correct?
1. It is a statutory body set up under the a. 1 only
National Commission for Minorities
b. 2 only
Act, 1992.
c. Both 1 and 2
2. Its mandate is to safeguard and protect
the interests of minorities provided in d. Neither 1 nor 2
the Constitution and laws enacted by
Parliament and the State Legislatures.
6) Microdots Technology sometimes seen in
3. It has powers to declare a community as
the news recently is used to?
a minority community.
a. Purification of sewage
Which of the statement(s) given above is/are
correct? b. Prevent accidents in roads
a. 2 only c. Replace Rat hole mining
b. 1 and 2 only d. Prevent theft of vehicles
c. 2 and 3 only
d. 1, 2 and 3

www.shankariasacademy.com | www.iasparliament.com
72

7) Al Jufra airbase sometimes seen in the news Which of the statement(s) given above is/are
recently is located in which of the following correct?
countries?
a. 1 only
a. Iran
b. 1 and 2 only
b. Libya
c. 2 and 3 only
c. Yemen
d. 1, 2 and 3
d. Saudi Arabia

Answers
8) Consider the following statements with
respect to Distributed Ledger Technologies 1. b
(DLT)  Indian Navy Ship LCU L-56, the sixth of the
1. It refers to technologies that involve the Landing Craft Utility (LCU) MK IV class
use of independent computers to ships, was recently commissioned into
record, share, and synchronize the Indian Navy.
transactions in their respective  LCU L-56 is an amphibious ship with its
electronic ledgers. primary role being transportation and
2. All virtual currencies use Distributed deployment of main battle tanks, armoured
Ledger Technologies (DLT). vehicles, troops and equipment from ship to
shore.
Which of the statement(s) given above is/are
correct? 2. a
a. 1 only  ‗Grain by Grain‘ report is the complete
assessment of the environmental performance
b. 2 only of the fertilizer industry in India.
c. Both 1 and 2
 Centre for Science and
d. Neither 1 nor 2 Environment along with its Green Rating
Project (GRP), prepared the report based on
42 parameters.
9) Sathyamangalam Tiger Reserve is located
 This is the 7th rating project undertaken by
in which of the following states?
GRP, which has been hailed as one of the most
a. Kerala credible and transparent instruments in India
for assessing the environmental performance
b. Telangana
of Indian companies.
c. Tamil Nadu
 GRP has, earlier, rated the pulp and paper,
d. Karnataka automobile, chlor-alkali, cement, iron and
steel and thermal power sectors.
3. b
10) Consider the following statements with
respect to Deepfake  The Union Government set up the National
1. It is a combination of deep learning and Commission for Minorities (NCM) under
fake. the National Commission for Minorities
Act, 1992.
2. It is an Artificial Intelligence software
that superimposes a digital composite  Section 2(c) of the NCM Act, 1992 itself clearly
on to an existing video or audio. states that a community is notified as
―minority‖ only by the government.
3. These are created by machine learning
models, which use neural networks to
manipulate images and videos.

www.shankariasacademy.com | www.iasparliament.com
73

 The National Commission of Minorities 6. d


(NCM) has no such jurisdiction to declare
 Microdot technology involves spraying the
minorities.
body and parts of the vehicle or any other
 The repository of such powers to declare a machine with microscopic dots, which give
community as minority lies with the Central a unique identification.
government.
 Use of this technology will help check theft of
4. d vehicles and also use of fake spare
parts.
 Also known by its scientific name Allium
Sativum, Kodaikanal Malai  The microdots and adhesive will become
Poondu (Kodaikanal Hill Garlic) is permanent fixtures/affixation which cannot be
known for its medicinal and preservative removed without damaging the asset, that is
properties. the vehicle itself.
 It has anti-oxidant and anti-microbial  The Ministry of Road Transport & Highways
potential, which is attributed to the presence of has recently issued a draft notification,
higher amount of organosulfur compounds, amending Central Motor Vehicle Rules.
phenols and flavonoids compared to other
garlic varieties.  It allows motor vehicles and their parts,
components, assemblies, sub-assemblies to be
 It is grown in the Kodaikanal Hills, affixed with permanent and nearly
Dindugul district, Tamil Nadu. invisible microdots that can be read
physically with a microscope and identified
 It‘s usually white or pale yellow and each bulb with ultra violet light source.
weighs 20-30g on an average.
7. b
 The Geographical Indications Registry has
recently granted the Geographical Indication  Libya's internationally-recognised
(GI) tag to Kodaikanal Malai government has said it has struck an airbase
Poondu (Kodaikanal Hill Garlic). used as a key staging post by forces loyal to
renegade military commander Khalifa
 According to the GI application, Kodaikanal Haftar in transporting troops and supplies.
Hill Garlic cultivation is done twice in a year,
once around May and for second time in  The air force struck a gathering of
November depending upon the suitability of mercenaries at the Al-Jufra airbase,
the climate. destroying a hangar for drones belonging to a
hostile country.
 The hill altitude, the misty condition and
the soil prevailing in the Kodaikanal region  Turkey and Qatar have been the main
are responsible for its medicinal property and supporters of the United Nations-backed
the long storage shelf life of the garlic. government, while Egypt, Saudi
Arabia and the United Arab Emirates
5. d (UAE) are backing Haftar.
 According to the Tiger Census 2018, Madhya 8. c
Pradesh saw the highest number of tigers at
526, closely followed by Karnataka at 524  Distributed Ledger Technologies
with Uttarakhand at number 3 with 442 tigers. (DLT) refers to technologies that involve the
use of independent computers (also referred to
 The survey, the fourth such since 2006, is a as nodes) to record, share, and synchronize
gargantuan exercise and conducted once in transactions in their respective electronic
four years. ledgers.
 India achieved its commitment to the  Keeping such distributed ledgers obviates the
St.Petersburg Declaration, of doubling need for keeping the data centralized, as is
Tiger population, much in advance to the 2022 done in a traditional ledger and all virtual
deadline. currencies use DLT.

www.shankariasacademy.com | www.iasparliament.com
74

 A transaction under DLT essentially refers to 2) Consider the following statements with
the transfer of ―value‖ from one to another. respect to Companies (Amendment) Bill, 2019
 Blockchain is a specific kind of DLT that came 1. It seeks to amend the Companies Act,
to prominence after Bitcoin, a cryptocurrency 2013.
that used it, became popular. 2. Under the Bill, any unspent annual CSR
 Cryptocurrencies such as Bitcoin use codes to funds must be transferred to one of the
encrypt transactions and stack them up in funds under Schedule 7 of the Act within
blocks, creating Block chains. six months of the financial year.

 It is the use of codes that differentiates 3. Under the bill, it has been made
cryptocurrencies from other virtual currencies. mandatory for companies to have
physical address and register to crack
 RBI Act has the enabling provisions to permit down on shell companies.
the central government to approve a ―Central
Bank Digital Currency‖ (CBDC) as legal tender Which of the statement(s) given above is/are
in India. correct?

9. c a. 2 only
b. 1 and 2 only
 The Sathyamangalam Tiger Reserve
(STR), Tamil Nadu has bagged the award c. 1 and 3 only
for highest increment of tigers in the last cycle
(the last four years) in the country. d. 1, 2 and 3

10. d
3) With respect to Henderson
Island sometimes seen in the news recently,
31-07-2019 consider the following statements:

1) Consider the following statements with 1. It is an uninhabited coral atoll.


respect to Muslim Women (Protection of 2. It lies at the centre of the South Pacific
Rights on Marriage) Bill 2019 Gyre.
1. It seeks to declare the practice of instant 3. It was included on the UN World
Triple Talaq as void and illegal. Heritage List in 1988.
2. It makes declaration of talaq a Which of the statement(s) given above is/are
cognizable offence, attracting up to correct?
three years‟ imprisonment with a fine.
a. 3 only
3. Under the bill, an affected muslim
woman is entitled to seek subsistence b. 1 and 3 only
allowance for her dependent children c. 2 and 3 only
but only for the first two children.
d. 1, 2 and 3
Which of the statement(s) given above is/are
correct?
a. 1 only 4) Which of the following is/are not the
applications of 5G technology?
b. 1 and 2 only
1. Reduce Pharmaceutical R&D costs
c. 1 and 3 only
2. Development of Robotics in surgery
d. 1, 2 and 3
3. Improved public safety with more
efficient trackers
4. Intelligent fleet and logistics
management

www.shankariasacademy.com | www.iasparliament.com
75

Select the correct answer using the codes given 8) Consider the following statements
below:
1. It is a designated Ramsar Site.
a. None
2. It is the largest brackish, humid tropical
b. 1 and 3 only wetland ecosystem located on the
southwest coast of India.
c. 2, 3 and 4 only
3. It supports the third largest waterfowl
d. 1, 2, 3 and 4
population in India during the winter
months.
5) 2021–2030 is observed by the United Identify the wetland that correctly matches
Nations (UN) as the decade of? with the above description:
a. Family Farming a. Point Calimere
b. Action on Nutrition b. Pallikaranai Wetlands
c. Ecosystem Restoration c. Kadulandi Wetlands
d. Eradication of Poverty d. Vembanad-Kol Wetlands

6) Which of the following languages 9) Which of the following is/are the challenges
is/are not mentioned in the eighth schedule of for 5G technology implementation in India?
the Indian Constitution?
1. Low Spectrum Fees
1. Nepali
2. Financial Stress faced by the telecom
2. Bodo service providers
3. Santhali 3. Fast pace of approvals from different
government agencies
4. English
4. Strengthening the security apparatus
Select the correct answer using the codes given with evolving technologies
below:
Select the correct answer using the codes given
a. 4 only below:
b. 1 and 4 only a. 1 and 2 only
c. 2 and 3 only b. 2 and 4 only
d. 1, 2 and 3 c. 1, 2 and 4 only
d. 1, 2, 3 and 4
7) Which of the following best describes the
term Kleptocracy?
10) Transiting Exoplanet Survey Satellite
 a. A Society without a publicly enforced (TESS) sometimes seen in the news recently
government or political authority belonged to which of the following space
 b. A system of governance where political agencies?
power is in the hands of a small class of a. NASA
privileged individuals
b. ESA
 c. Rule by the proletariat, the workers, or the
working class c. JAXA

 d. A society whose leaders make themselves d. ROSCOSMOS


rich and powerful by stealing from the rest of
the people

www.shankariasacademy.com | www.iasparliament.com
76

Answers nutrients into the waters surrounding


Henderson to feed huge colonies of sea birds.
1. b
 The atoll's ecosystem is so rich that Henderson
 The bill seeks to declare the practice of instant was included on the UN World Heritage
Triple Talaq as void and illegal and makes it a List in 1988, with the body hailing it as an
cognizable offence. untouched paradise.
 It also provides allowance to women victims 4. a
and their dependent children.
 5G technology promises accelerated
 The offence is made compoundable with the download and upload speeds.
permission of the Magistrate at the instance of
the married Muslim woman upon whom talaq  Network latency, or the lag in data
is pronounced. communication between wireless devices, is
minimal on 5G networks.
2. d
 Also, 5G is a more versatile technology and can
 The Companies (Amendment) Bill, 2019 was therefore be operated over three different
recently passed by the Parliament. spectrum bands, resulting in superior coverage
 It amends the Companies Act, 2013. and reception.

 Under the Act, if companies which have to Applications


provide for CSR, do not fully spent the funds, Health Care
they must disclose the reasons for non-
spending in their annual report.  Reduce Pharmaceutical R&D costs

 Under the Bill, any unspent annual CSR funds  Video/3D Video appointments for non-life
must be transferred to one of the funds under threatening ailments
Schedule 7 of the Act (e.g., PM Relief Fund)  Development of robotics in surgery
within six months of the financial year.
 More efficient health and fitness monitors
 To crack down on shell companies, the
minister said it has been made mandatory Smart Cities and Homes
to have physical address and register to
 3D Video calls and holograms
ensure such companies exist on ground.
 Innovative augmented and Virtual Reality
3. d
applications
 Henderson Island is an uninhabited coral
 Improved public safety with more efficient
atoll that lies almost exactly halfway between
trackers
New Zealand and Peru, with 5,500 kilometres
(3,400 miles) of the ocean in either direction.  Remote device control
 Despite its extreme isolation, a freak Transport
confluence of geography and ocean currents
means Henderson has one of the highest Connected vehicles which allow data collection from
concentrations of plastic pollution on traffic signals and road-side sensors will lead to
the planet.  Reduced congestion
 Henderson lies at the centre of the South  Intelligent fleet and logistics management
Pacific gyre, a vast circular ocean current
that runs anti-clockwise down the east coast of Autonomous cars with little or no help from humans
Australia and up the west coast of South can
America.  Improve road safety
 The gyre should be a boon for the 10-  Increase mobility
kilometre-by-five speck of land, carrying rich
5. c

www.shankariasacademy.com | www.iasparliament.com
77

6. a  Strengthening the security apparatus with


evolving technologies
 The Eighth Schedule to the Constitution
consists of the following 22 languages:- 10. a
 (1) Assamese, (2) Bengali, (3) Gujarati, (4)  Transiting Exoplanet Survey Satellite
Hindi, (5) Kannada, (6) Kashmiri, (7) Konkani, (TESS) is a space telescope for NASA‘s
(8) Malayalam, (9) Manipuri, (10) Marathi, Explorers program
(11) Nepali, (12) Oriya, (13) Punjabi, (14)
Sanskrit, (15) Sindhi, (16) Tamil, (17) Telugu,  It was launched in 2018 to detect small planets
(18) Urdu (19) Bodo, (20) Santhali, (21) with bright host stars in the solar
Maithili and (22) Dogri. neighbourhood.

 Of these languages, 14 were initially included  The spacecraft will be looking for a
in the Constitution. phenomenon known as a transit, where a
planet passes in front of its star, causing a
 Sindhi language was added in 1967. periodic and regular dip in the star‘s
brightness.
 Thereafter three more languages viz., Konkani,
Manipuri and Nepali were included in 1992.  It will do an all-sky survey from an orbit
between the Earth and the moon.
 Subsequently Bodo, Dogri, Maithili and
Santhali were added in 2004.  Three new planets, which orbit a star situated
73 light years away from the Earth, have been
 Union Culture Minister has recently discovered using TESS, NASA's planet
announced the inclusion of Nepali and hunting satellite.
Santhali languages in the Scheme for grant of
Senior/Junior Fellowships of Culture
Ministry.
7. d
8. d
 Vembanad Lake, a designated Ramsar site
and one of the most polluted waterbodies in
the world.
 It is the largest brackish, humid tropical
wetland ecosystem on the southwest coast of
India, fed by 10 rivers and typical of large
estuarine systems on the western coast,
renowned for its clams and supporting the
third largest waterfowl population in India
during the winter months.
9. b
Challenges for 5G implementation in India
 High Spectrum Fees
 Financial Stress faced by the telecom service
providers
 Slow pace of approvals from different
government agencies
 Permission from multiple local authorities to
use public infrastructure for installing small
cell network

www.shankariasacademy.com | www.iasparliament.com

Você também pode gostar